Conference Notes 9-16-2015

Williamson/Parker     Oral Boards

 

Case 1.  50 yo male with respiratory distress.  HR=100 BP=140/80RR=34T=98.7   Patient became SOB at subway station.   Exam reveals copious oral secretions. Patient also vomited and had abdominal cramping. EMS notes that other persons in the subway have similar symptoms.

 

*Cholinergic Toxidrome

Diagnosis is cholinergic toxidrome due to Sarin (organophosphate gas).   Treatment is high dose atropine until respiratory secretions have improved significantly, intubation,  and 2PAM.  Consult poison control.   Patient needs to be decontaminated. Staff needs to use PPI to prevent secondary exposure to toxin.  Patient’s clothing can off–let toxin for 30 minutes.  Sarin is colorless and odorless.

 

Elise comment: I saw a patient die of organophosphate poisoning in Africa.  It was horrible. The patient basically drowned in his own secretions because we did not have enough atropine.    Andrea comment: Initial dosing of atropine is 2-6 mg and go up from there until secretions are dried up.   There are stockpiles of atropine around the city and country.   The poison center has access to these stockpiles.  You have to get 2PAM into the patient as soon as possible to prevent irreversible aging of the acetylcholinesterase.

 

Case 2. 16 yo male with a headache for the last 10 days.  Vitals are normal.   Patient notes a weird feeling and paresthesias from waist down.  Legs have spasms occasionally.  Bedside ultrasound shows urinary retention.  MRI of the spine shows transverse myelitis. 

 

Transverse_myelitis_MRI.jpg

*transverse myelitis

 

Diagnosis is transverse myelitis due to multiple sclerosis.  Treatment is IV steroids.  Patient required foley catheter drainage of urine.   Transverse myelitis is an inflammation of the spinal cord.

 

In most cases a sensory level is documented, most commonly in the mid-thoracic region in adults or the cervical region in children. Pain in the back, extremities, or abdomen is also common while paresthesias (e.g., tingling, numbness, burning sensations) are typical in adults. Sexual dysfunction is also the result of sensory and autonomic involvement. Increased urinary urgency, bowel or bladder incontinence, difficulty or inability to void, and incomplete evacuation of bowel or constipation are other characteristic autonomic symptoms. Spasticity and fatigue are other symptoms common to transverse myelitis. Additionally, depression is often documented in TM patients and must be treated to prevent devastating consequences.  (Transverse Myelitis Association)

 

Case 3.  47 yo male presents with right foot pain for one week.  Vitals normal except for heart rate of 102.   Patient states that the pain began after a heavy board fell on his foot.   X-rays show lis franc injury.

 

*Lis Franc injury

Treatment requires ORIF and non-weightbearing for 6 weeks.   Harwood comment: this is a pretty severe lis franc injury.  Lis franc injuries usually involve the first and second metatarsals.  This xray shows all the metatarsals dislocated. 


 

Levato     Community Acquired Cellulitis


For uncomplicated cellulitis with no abscess use either cefazolin or clindamycin.   Elise raised concerns about the level of clindamycin resistance at our institution.   Harwood added that he would prefer to give Bactrim if you suspect MRSA.  

Levato’s Bottom line:  use more ancef and less vanco.  Patients without comorbiditities who have straightforward cellulitis with no fluctuance have a strep cellulitis. 


*Algorithm for soft tissue infections (Thanks Elise)


 

Remke      M & M


In order to respect the confidentiality of M&M, I am not going to give case details of M &M’s.  I will just present the take home points.


Be sure you have ordered a type and screen on pregnant patients who are bleeding.  If they deteriorate  and need transfusion, waiting for the type and screen can cause delay or necessitate the use of O negative blood.


Quantify vaginal bleeding based on whether it is more or less than a normal period.

On pelvic exam, if there is clot or tissue in the os, remove it.  Removing tissue and clot from the os can help the uterus to contract and lessen bleeding.


*Liklihood of Ectopic Pregnancy

 

In the setting of pregnancy, consult OB service for heavy bleeding, open cervical os, and positive beta hcg with no iup seen on ultrasound. 

 

Things to tell patients:

25% of pregnancies have bleeding

If a viable fetus is seen on ultrasound the risk of miscarriage is less than 10%.

Miscarriage is more likely with heavy bleeding, bleeding lasting a week or more, and if pain is associated with bleeding.

 

Emergency Physician Cognitive Issues:

Pay attention to subtle vital sign changes

Double check that the labs you need are ordered like type and screen.

Listen to your nurses when they raise a concern.

Get help from consultants when you are faced with a difficult situation.

 

Kelly Williamson comment: RH negative Patients who have a subchorionic hemorrhage on ultrasound but are not bleeding into the vagina do not need rhogam

 

Ortiz-Romero     Imaging the Pediatric Acute Abdomen

 

On ultrasound, a normal appendix is smaller than 6mm, compressible and has no surrounding free fluid.  Appendicitis is larger than 6mm, non compressible and hyper-vascular.  There will be point tenderness over the appendix with appendicitis.  Unfortunately 70% of the time the appendix is not visualized with ultrasound.

 

If you have an Alvarado score less than 5 and have an ultrasound with no inflammatory changes without visualization of the appendix, that clinical scenario has a negative predictive value of 99% for appendicitis!

 

Value of Focused Appendicitis Ultrasound and Alvarado Score in Predicting Appendicitis in Children: Can We Reduce the Use of CT?

Blitman NM1, Anwar M, Brady KB, Taragin BH, Freeman K.

Author information

  • 11 Department of Radiology, Jacobi Medical Center, Albert Einstein College of Medicine, 1400 Pelham Pkwy S, Bronx, NY 10461.

Abstract

OBJECTIVE:

The purpose of this study was to evaluate the effectiveness of focused appendicitis ultrasound combined with Alvarado score to accurately identify appendicitis in children in whom it is suspected, thereby reducing unnecessary CT examinations and associated radiation exposure.

MATERIALS AND METHODS:

We retrospectively evaluated the focused appendicitis ultrasound, CT, clinical, and laboratory findings of 522 consecutively registered children (231 boys, 291 girls; mean age, 13.04 [SD, 5.02] years; range, 0.74 months-21 years) who underwent focused appendicitis ultrasound for abdominal pain in a pediatric emergency department from January 2008 through October 2009. All children underwent surgery or clinical follow-up to exclude missed appendicitis. Sonographic findings were characterized as positive, negative, or inconclusive (appendix not visualized). Alternative diagnoses were noted. Alvarado score (0-10 points based on multiple clinical criteria) was determined. Focused appendicitis ultrasound and Alvarado score results were compared with surgical and pathologic reports.

RESULTS:

Both focused appendicitis ultrasound results and Alvarado score were associated with likelihood of surgery for appendicitis (p = 0.0001). Focused appendicitis ultrasound had conclusive results: 105 positive and 27 negative in 132 of 522 (25.2%) children. In the 390 of 522 (74.7%) children with inconclusive focused appendicitis ultrasound findings, 43 of 390 (11.0%) eventually had a diagnosis of appendicitis with CT (n = 26) or Alvarado score (n = 17). Among children with inconclusive focused appendicitis ultrasound findings and an Alvarado score less than 5 (241/522, 46.1%), only one patient had appendicitis. The negative predictive value (NPV) of inconclusive ultrasound findings and low Alvarado score combined was 99.6%. Among children with inconclusive focused appendicitis ultrasound findings and an Alvarado score of 5-8, the NPV decreased to 89.7%.

CONCLUSION:

Children with inconclusive focused appendicitis ultrasound findings and a low Alvarado score are extremely unlikely to have appendicitis (NPV, 99.6%). Avoiding unnecessary CT of these patients is a safe approach to diagnosis.

 

When using CT for appendicitis, you need some peritoneal fat to identify the appendix.  So, very thin kids will be difficult to visualize the appendix on CT because they lack peritoneal fat.   However if there are no secondary signs of inflammation in the area of the appendix, that also makes appendicitis unlikely even if the appendix is not visualized.   To limit radiation exposure, you only need to do a pelvis CT.  Give IV and rectal contrast if the patient can tolerate them.   The cutoff for a normal appendix on CT is 7mm.

 

MRI can also be used to diagnose appendicitis. We are currently testing a MRI protocol for appendicitis.   The patient first gets scored on the pediatric appendicitis score.  If the score is 2 or more they are eligible for MRI testing.

 

Ultrasound is the first line test for identifying intussusception. 

 

Girzadas      ACGME Resident Survey Results

 

C Kulstad/Lovell    Preventing Diagnostic Error     Cognitive Bias

 

Misdiagnosis is the most common medical mistake and is a leading cause of malpractice claims. 

 

“Everyone is a human first and a novice or expert second”

 

There are 2types of causes for diagnostic error.  There are systems error and cognitive error.

 

 

*System and Cognitive Error

 

Error is more common when: uncertainty is high, the patient is unfamiliar to the physician, and the disease has an atypical presentation.   Cognitive error is a risk in high-pressure environments with many distractions. (the ER).

Nurses when acting as the wingman to the physician also need to be aware of their biases.  The nurse has the potential bias the physician either towards or away the right diagnosis.  If a nurse is biased against some aspect of the patient it can affect how the whole team approaches the patient. 

 

System 1 and System 2 Thinking.  To be a good EM Clinicianyou have to be able to switch back and forth between both systems.  Experienced clinicians have better developed system 1’s.  They have the experience to know when to switch into system 2 thinking.

 

*System 1 and System 2

Heuristics use pattern recognitions or mental shortcuts to solve problems.   Bias impacts heuristics a great deal.   This is a problem because ER docs use heuristics all the time in an environment that is prone to bias the heuristics.

 

Croskerry Six Biases:

1.     Over attachment to a particular diagnosis.  Anchoring on a specific feature too early and failing to adjust.

2.     Inheriting someone else’s thinking.  Following the cognitive path initiated by another provider.  This bias makes sign out a dangerous time.

3.     Failure to consider alternatives.  Tendency to stop looking once something is identified.

4.     Errors in estimation of prevalence.  Diagnoses that readily come to mind may be used more frequently.

5.     Bias related to patient characteristics or context.  Basically the physician has a personal perception about the patient and approaches the patient based on that concept of the patient.

6.     Errors associated with the clinician’s personality.  Over/under confidence is one example.   We need to have the appropriate level of both confidence and humility.

 

Tactics to overcome bias:

Cognitive debiasing:  Consider alternatives. Use metacognition or think about how you are thinking about a patient. 

Structured handovers:

Checklists :  Checklists help us to think of options we did not initially consider

QV&V: Qualify the source, validate the information, and verify the information.  Basically check that the info you are basing your decision on is correct and valid.

STEP Thinking: Story, Testing, Evaluate, and Plan

EMR Enhancements

Conference Notes 8-5-2015

E. Kulstad         Research Update

ACMC EM is #1 in the world in enrolling patients with intra-cerebral hemeorrhage  and hypertension into the ATACH 2 Trial!  Please continue to enroll these patients in the study.

The Macy Catheter can be used to give almost any medication rectally.   You can give up to about 500ml of water or pedialyte per hour rectally. This may be a therapeutic bridge option in the person who has a difficult IV access.

C. Kulstad       Study Guide Cardiology

There is no mortality benefit to routinely taking non-STEMI, ACS patients to the cath lab.   Thus, the patient with chest pain and bumped troponins and a non-STEMI EKG gets equal benefit from medical management as they would from going to the cath lab.

The absolute mortality risk reduction of ASA in ACS is 4%.  This data is derived from the 1988 ISIS study that showed ASA brought mortality down from 13% to 9%.  The absolute mortality reduction from heparin and LMWH in non-STEMI ACS is 0%.   The data for heparin and LMWH in STEMI is not known.  There is no Cochrane review on this topic.  We don’t really know the mortality benefit, if any, for heparin in STEMI.

POISE-2 study showed that you should hold ASA prior to abdominal surgery.  There is no increase in mortality if you hold ASA prior to abdominal surgery in patients with prior MI.  If you give ASA prior to surgery there is an increase in bleeding.  Elise comment: Most surgeons should not have a problem limiting surgical bleeding due to a patient’s ASA use.

electrical alternans

*Electrical alternans is a poorly sensitive and reasonably specific finding of pericardial effusion.

Stress echo is a moderately sensitive (80%) and specific (80%) test for coronary stenosis.   It however is not a good test to identify risk for major adverse coronary events (MACE)

A murmur at the right 2nd intercostal space is consistent with aortic stenosis.  These patients can present with chest pain, dyspnea and exertional syncope.   It is a murmur that is easy to miss.  The prevalence of aortic stenosis in the elderly is 3% (pretty high prevalence).  There is now a new option of transcatheter aortic valve implantation that allows for valve replacement therapy without thoracotomy.

pacemaker indications

*Indications for temporary and permanent pacemaker

Hi dose NTG has shown mortality benefit in CHF.   We probably under dose IV NTG.   One strategy to get a higher dose of NTG into a patient is to use sublingual spray.  Each spray gives 400micro grams per spray.  You can give a spray every few minutes in the hypertensive, pulmonary edema patient.

Tobacco cessation is the single most profound risk reduction strategy in patients with CAD.

Pericarditis pain is worse lying down.  Patients can also have pain with swallowing.

Burns/C. Kulstad      Oral boards

Case 1.  24 yo male presents with agitation and hyper-adrenergic toxidrome. Patient had to be restrained by EMS personel and was fighting against restraints.   His troponin and AST are elevated.   His CK is markedly elevated.  Urine tox screen is positive for cocaine.  Patient management of cocaine overdose required relatively high dose IV benzo’s for agitation and hyperadrenergic vital signs. Treatment for cocaine toxicity is benzodiazepines as your  first/second/third line.  Patient also required  IV fluids for rhabdomyolysis.  A bicarb drip for cocaine-induced rhabdomyolysis is controversial.  Cooling measures were also indicated to manage hyperthermia. Avoid beta-blockers in cocaine toxicity.  You can get unopposed alpha effects.

Case 2.  28 yo female with left shoulder pain.

ac separation

*Image

Diagnosis was AC joint separation.   Treatment is sling, pain control, and non-emergent orthopedic referral.  You have a two-week time frame to surgically repair an AC separation.   AC joint separation is the most common shoulder injury in athletes. High-grade separations require surgical repair.

Xray findings of AC separation: The acromion is not level with the lateral clavicle.

ac sepatration xray

*AC separation Xray

Case 3.  20 month old female brought in for crying.

svt peds

*Monitor strip

Exam shows enlarged liver.   Patient has cool extremities.

Diagnosis was SVT.  Treatment was IV adenosine 0.1mg/kg initial dose.  The rhythm slowed temporarily but then returned to SVT.  Second adenosine dose was 0.2mg/kg, which also did not convert the patient.  Child appeared more clammy/shocky so the next move was synchronized electrical cardioversion.  Patient was given intransal fentanyl prior to cardioversion.  Initial shock was 0.5j/kg.  Patient converted with first attempt at cardioversion.   If first attempt fails you can go to 1j/kg.

Remke    Pediatric SVT

Case 1.  7 day old with a heart rate of 300.   BP 75/47.   Afebrile.   Patient had been feeding well.  Lungs were clear.  Color was good.  Rythm Strip showed SVT.  Child was treated by peds cardiology in the ED with propranolol and rate was controlled.

Snip20150806_1.png

 *3 Types of SVT

To diagnose SVT  in pediatric patients, the heart rate should be great than 220 in an infant and >180 in a child.   There should be no P waves on EKG.  The rhythm should be very regular with no variability.  Sinus rhythm has more variability than SVT.   Notching of the T wave suggests sinus rhythm with the P wave buried in the T wave. 

svt vs sinus better

*Sinus Tach vs. SVT

svt algorithm

*Pediatric SVT Management Algorithm

Vagal maneuvers include: ICE to the face, Valsalva maneuver, and rectal stimulation with a thermometer.

For known WPW use procainamide instead of adenosine.  Elise comment: For orthodromic narrow complex SVT it is ok to use adenosine even if it turns out to be WPW.  It is the AHA recommendation. 

Avoid verapamil in infants and children.  It is contraindicated in patients under 1 year of age because it can result in V-Fib and hypotension.   It can be used as a second or third line therapy for SVT in teenagers.

After cardioversion, admit infants and discharge older kids/teenagers after a 2-4 hour period of observation.  If you identify WPW discuss with cardiology about admission for ablation.

Ede      Low Risk Chest Pain

Chest pain patients are considered low risk if the probability of major adverse cardiac events is 1-2%.

Patient’s response to GI cocktail or NSAID or NTG is not reliable for ruling in/out ACS.

Women may have more atypical symptoms.

The-HEART-Score.png

*HEART Score        Patients who are low risk by HEART score and have 2 negative troponins 3 hours apart have a less than 1% risk of MACE  (major adverse cardiac events)  in 6 weeks.

feb-2015-cp-shared-medical-decision-making-5-1.png

*Heart Score Decision Making

Hart        Tachyarrhythmias

Adult tachycardia algorithm

*Tachycardia Algorithm.   I would only add that procainamide can also be used and is probably preferred for wide complex irregular tachyarrythmias (WPW).

If you have IJ central line access, you can use a lower dose of adenosine because of a faster transit time to the heart.  Braden Parker noted that 3mg of adenosine worked quite well in a patient with SVT that he treated through an IJ line. 

Adenosine causes a sense of doom or chest pain in patients.  You should give some sedation prior to administering adenosine.

wpw

*WPW Syndrome

Avoid AV nodal blocking agents (betablockers, calcium channel blockers)  in antidromic (wide complex) tachycardias.  Blocking the AV node in antidromic tachycardia can result in death.

Aflutter usually has a rate right around 150.

a-fib-and-a-flutter-venn-diagram.jpg

*Atrial Fibrillation and Atrial Flutter

Calcium Channel Blockers have been shown to be superior to beta-blockers for Afib.

junctional tachycardia_digoxin

*Junctional Tachycardia can be hard to diagnosis.  Consider digoxin toxicity when you see Junctional Tachycardia.

VT

*Diagnosing V-Tach

Snip20150806_2.png

*Girzadas note: Another Approach to Diagnosing V-Tach on ecgpedia.org. I have always struggled with differentiating VT and SVT with Aberrancy.  This seems like a simple way to do it.  In lead II if the initial upstroke or downstroke of the QRS takes more than a box it is likely VT.  A caution, It is not validated yet. 

First line treatment for Stable Ventricular Tachycardia is Procainamide.   Second line is Amiodarone.

torsades

*Torsades      Treat with Magnesium or Overdrive Pacing

Parker         PodCast Pearls  for Codes

Traumatic Arrest is usually due to one of a few things: exsanguination,  pericardial tamponade, or tension pneumothorax.    Based on that, is there any reason to do CPR for traumatic arrest?  CPR can also hinder other activities/procedures being performed on a trauma patient. CPR can increase the risk of blood exposure to the resuscitationist.

Mnemonic for PEA in the Trauma patient:  HOTT=  Hypovolemia, Oxygenation, Tamponade and Tension Pneumothorax.

C Kulstad and Girzadas comments:  We are uncomfortable dropping CPR completely from trauma resuscitation.  We think there is medical-legal risk to that.  Possibly a more middle of the road approach would be to prioritize life saving procedures.  If CPR is hindering other life saving procedures such as gaining IV access, relieving tamponade,  or relieving tension pneumothorax stop CPR for as short a time as possible and perform the procedure and re-start CPR as soon as possible.

PEA Management:  Use ultrasound to identify causes.  If the QRS in PEA is wide think metabolic causes like hyperkalemia, TCA overdose, infarction.  If the QRS is narrow think obstructive causes such as tamponade, tension pneumo, or PE.

Jamieson        Parasites

A parasite lives at the expense of the host.

The CDC Lists 5 Parasitic Infections we should know.

Chaga’s disease: Caused by protozoan Trypanosomi Cruzi.  Acutely, patients have fever and malaise.  They can also acutely have eyelid swelling, myopericarditis, and meningitis.  Chronically patients can have CHF or megaesophogus.   You need to get the treatment drugs from the CDC.

chagas-disease-bug.jpg

*Eyelid edema from acute Chaga’s Disease

Cystercercosis:  You get the worm from eating infected pork. Neurocystercercosis can result in seizures.   When the infection is in the muscle if feels like bumps.   Treat with albendazole, anti-seizure meds, steroids, and surgery.

neurocystercircosis

*Nueorcystercercosis

Toxocariasis: Caused by ingestion of cat or dog feces. Acutely the patient will get a hepatitis or pneumonitis.  The pneumonitis can look like asthma.  Kids can also get unilateral visual impairment from fundal involvement.

Toxoplasmosis: Caused by ingesting cat feces.   Infection can be spread vertically from mom to infant.  Congenital toxoplasmosis can cause spontaneous abortion, fetal death, and fetal neuro deficits. Immunocompetent adult patients can have a mono-like syndrome.  Immunocompromised adult patients can get encephalitis.  

Trichomoniasis:  Most common non-viral STD.  Can be asymptomatic for months.  Women have vaginal symptoms.  75% of men are asymptomatic.  Can result in pregnant women having premature rupture of membranes and preterm delivery.

Conference Notes 7-29-2015

If you don't see images scroll to the bottom and click "Read in Browswer"

Paquette      Study Guide     Special Patients

Black tar heroin mixed with lemon juice may harbor fungus that when injected can result in fungal chorioretinitis.

50% of elderly patients hospitalized after a fall die within a year.   Falls represent a sentinel event and are not a normal part of aging.

trench-foot-pictures-6.jpg

*Trench Foot is stratum corneum breakdown due to continuous water exposure.  This is commonly seen in homeless persons forced to wear wet shoes and socks for prolonged time periods. Treatment is dry socks and dry shoes.

Treat maggots with ethyl chloride.  The topical Freezing Spray (ethyl chloride spray) works well to kill maggots.   A recent study showed that a yankaur suction device works well to remove maggots.

To optimally visualize the airway in obese patients, use RAMP positioning.  The goal is to line up the external auditory meatus with the sternal notch. 

ramp2.png

*RAMP positioning

Cotton fever is due to a gram negative rod that exists on cotton balls. This gram negative rod can get into heroin when the cotton is used as a filter.   The fever is due to an exotoxin from the gram negative rod and is self-limited.  No treatment other than antipyretics is indicated for cotton fever.  However, this provides little help because it will be hard to differentiate cotton fever from sepsis when the patient first presents to the ED.

The right heart valves are more prone to endocarditis than left side heart valves.  Injection cocaine use is more likely to cause endocarditis than injection heroin use because cocaine has a shorter half life resulting in more injections and cocaine causes direct damage to the valves.

Obese patients tend to have EKG’s with low voltage and inferior t-wave flattening/inversions.  These changes are thought to be due to increased chest wall thickness and possible shift of the normal axis.

Trauma patients over age 50 account for 30% of Trauma deaths.  Trauma patients of advanced age have less physiologic reserve to withstand severe injury. 

Subdural hematoma is more common in elderly patients.

sub vs epi

*Subdural vs. Epidural Hematomas

In patients over 80 years of age, chest pain is not the most common symptom of AMI.  Dyspnea and weakness are more common presenting complaints.

Downs syndrome patients have chronic atlanotaxial instability

aa instability

*Atlanto-axial Instability

“Parents have brought a child with Down’s Syndrome ‘walking funny’ to an emergency department for evaluation,” Dr. Bull recalled. “And then the ED would X-ray the child’s hips and say nothing is wrong. But they were not aware of the importance of evaluating the cervical spine.”

The neurologic manifestations of symptomatic AAI include easy fatiguability, difficulties in walking, abnormal gait, neck pain, limited neck mobility, torticollis (head tilt), incoordination and clumsiness, sensory deficits, spasticity, hyperreflexia...and (other spinal cord) signs and symptoms. Such signs and symptoms often remain relatively stable for months or years; occasionally they progress, rarely even to paraplegia, hemiplegia, quadriplegia, or death. Trauma rarely causes the initial appearance or the progression of these symptoms. Nearly all of the individuals who have experienced catastrophic injury to the spinal cord had weeks to years of preceding, less severe neurologic abnormalities..."

During intubation of a Down’s patient, Great care must be taken to maintain the

neck in a neutral position. This may be accomplished

by placement of a soft collar after induction of anes-

thesia to avoid extreme neck flexion, extension, and

rotation. Leaving the collar in place may also serve as

a valuable reminder to all caring for the patient dur-

ing the perioperative and postoperative period that

cervical instability may exist

autonomic dysreflexia

*Autonomic Dysreflexia can be seen in spinal cord injured patients who have a uti, constipation, pressure ulcer or constricting clothing below the spinal cord level of their injury.  The patients don't feel discomfort from these problems but develop HTN, flushing, sweating, and headache. 

Asokan     Interpreting the Chest Xray

We are responsible for everything on the X-ray, not just the aspects that we are interested in.  Be sure to look at the neck, axillae, and abdominal areas of a CXR.

Pathologic densities in the lung show up as a white density signifying abnormal fluid.

Normal-CXR-Labelled.jpg

*Normal CXR

If you see an abnormality, look for a second or third less obvious abnormality.  Your eyes get stuck on the most obvious abnormality.

Consolidation is when the alveolar space is filled with inflammatory exudate.  Consolidations have air bronchograms.

Consolidation1_big.jpg

Consolidation with air bronchograms

pleural effusion

*Upright film shows a pleural effusion with a curved upper meniscus.

When looking for pneumothorax, look between the ribs for the pleural line.

cxr_ptx_1.png

*Pneumothorax. Note the pleural line between the ribs on the right.

CHF

*CHF Findings

peribronchial cuffing

*More views of peribronchial cuffing

sbo

*Small Bowel Obstruction

coffee bean

*Coffee Bean Sign of Volvulus

Jamieson     M&M

Life Lesson #1:  Don’t use dermabond around the eye.   If you need to disolve glue on an eyelid or your finger gets glued to the skin,  use erythromycin ointment.   It doesn’t break glue down immediately but it will over a relatively short time.

Life Lesson #2:  In order to figure out  what is going on with non-verbal nursing home patients, it may require a call to the nursing home.   Steve described the case of a nonverbal nursing home patient who otherwise was well appearing. He called the nursing home and learned that the patient was normally quite talkative. He proceeded to CT scan the patient and identified an intra-cerebral hemorrhage.

Life Lesson #3: Beware the back hallway bias.  This anchoring bias can cause you to downplay a patient’s acuity and symptoms.   Just because a patient is in the hallway and not in a critical care room doesn’t mean they can’t have a critical illness.

Life Lesson #4: Intubating obese trauma patients can be risky.  Expect O2 desaturation to occur more quickly than expected.  Be sure you have maximized your  pre-oxygenation.  Place the patient in RAMP position.  Use high flow continuous nasal oxygenation during the pre-oxygenation and intubation periods.  Have video laryngoscopy ready as either your first device or back up device.  Have an LMA ready to use as a rescue device.   Be sure to check that your oxygen source is working properly.  You can use ketamine as an induction agent to perform delayed sequence intubation; you can preoxygenate and evaluate the airway prior to neuromuscular blocking.

Htet/Meyers/Faculty       Orthopedic Exam Lab

Conference Notes 7-22-2015

A special Thank you to Christine Kulstad for her help with the Conference Notes this Week!

Lovell/Htet        Oral Boards

Critical Actions Case 1:  Septic Shock from Capnocytophaga canimorsus/dog bite

--Identify Sepsis

--Rapid IVF, early broad spectrum antibiotics including dog bite coverage

--Elicit history of dog bite

--Reassessment

--ICU admission

(also consider Xray, irrigation of wound, consider tetanus update, rabies)

Teaching points:  Capnocytophaga canimorsus

--fastidious GNR identified in 1976, likes to eat iron

--normal oral flora of kitties and doggies

--Risk groups:  asplenic, alcoholics, beta thal, smokers (high iron), immunosuppression

--Alcoholics have high blood iron + immune comp.

--WORST OUTCOMES in asplenics:  high iron + decreased phagocytosis; high M/M!!

--Symptoms usually within several days

--Fever, V/D, malaise, abdominal pain, myalgia, confusion, dyspnea, headaches, rashes

--Special culture media

--Tx:   Dogmentin!  so use IV Unasyn, also susceptible to third gen. Cephalosporins

Critical Actions Case 2:  Submassive to Massive PE

--Recognize risk for PE (travel)

--Diagnose PE

--Risk stratify (Echo, BNP, Trop)

--Recognize decompensation and need for thrombolytics

--Assess contraindications to tPA

--ICU admission, Intensivist consultation

Teaching points:   Submassive/Massive PE

--Submassive:Normal BP but RV dysfxn (Echo or BNP) or myocardial necrosis (Trop)

--Massive: Low BP for 15 minutes or code or resp. failure or need for pressors

--Thrombolytics for massive, case by case for submassive

--r-tPA dose 100 mg IV over 2 hours

--Code dose, 50 mg bolus (maybe + 50 mg)

--start IV Heparin when PTT < 2X nml

--Lytics in PE:  2% ICH risk, 6% major bleed/transfusion

Critical Actions Case 3:  Jellyfish envenomation

--Identify as Jellyfish envenomation

--Irrigate 

--Consider vinegar, hot water

--Monitor for systemic sx, allergic sx

--Treat with benadryl, consider steroids, analgesics

Teaching points: Jellyfish Envenomation

--Mechanism-tentacles with venom containing nematocysts

--Symptoms--Itch, swell, burn, tingling/numbness.  Severe with N/V, HA, weak, dizzy, SOB

--Treatment-irrigate with NS or sea water (fresh water makes worse), remove possible remaining nematocysts by shaving skin or scraping skin

--Use benadryl, consider steroids, give analgesics

--vinegar-prevent further nematocyst firing, hot water to breakdown venom-some controversy over effectiveness

Levato     Pharmacy Update

Community-acquired but complicated UTI- E coli, Klebsiella, Proteus most likely. We have been using ceftriaxone for this but it is a broad spectrum antibiotic that is causing c diff in our hospital. Cefazolin found to be 89% sensitivity in E coli, 90% in Klebsiella, 100% Proteus. So new guidelines are to use cefazolin for these patients. CAUTION- not appropriate for nursing home patients. OK for pyleonephritis.

Ketaneh  (Class of 2014)  Starting out as a New Attending

Looking for a job: Take some pressure off- its just your first job, you can change it later. He interviewed all over and recommends it. Prospective employers will usually pay for your interview expenses so don’t shy away from multiple interviews. It helps to see a lot of practice environments too, see where you fit in.  Use the faculty resources here (they know somebody everywhere).

You will have a lot more money- try to live like a resident and save for a while. Student loans will come due, 401k should be funded. Need to find your work-life balance- how many shifts due you want to work to pay for your lifestyle.

Starting a new job- get familiar with the EMR before your first real shift. Will save you tons of time and pain before your first shift.

Have your charting macros prepared before you start.

Know where you are going to look stuff up.

Be super nice and respectful to everyone.

Get your charting done so you stay off the medical records radar. 

Get to know the culture of your workplace: protocols, antibiotic choices, sedative choices.

Nick told a scary story of how he ordered a fentanyl drip and the pharmacists and nurses were not familiar with using fentanyl as sedation and dosing errors were made.

At most ED’s you at times will be working alone with a couple of nurses.

Be very clear when communicating with nurses, they will do exactly whatever you say.

An important part of our job is charting and billing.  It’s just a fact of life.  Your income and job security depend on it.

ups and downs

*Ups and downs.   You have to be able bounce back from the downs as an ER doc and temper/be humble with the Ups

There are other jobs out there.  You can change jobs.   It is important to protect your reputation so people think well of you and you keep the door open to other opportunities.

Be smart with your money.   It is OK to rent for awhile until you have a clear idea of your monthly income.  Then live below your means so you are funding your retirement properly.

As a resident and attending, going the extra mile will not go unnoticed.

Get your paper work done. 

Favorite FOAM websites:  lifeinthefastlane.com, aliem.com, emcrit.com

The goal for an ED doc in the community is to not be ultra cutting edge and not be practicing medicine from 20 years ago.  You want to be somewhere in the middle and up to date. 

Regan       Seizures

causes of seizures

*Causes of Seizures

Generalized Seizures: Affects both Cerebral Hemispheres. There is loss of consciousness.

siezuretypes.gif

*Seizure Types

Partial Seizures include: Simple in which conscious is maintained and Complex in which consciousness is altered.

Initial questions to ask the patient:  Do you have a seizure disorder?     Is there anything that would have lowered you seizure threshold?  (sleep deprivation, fever, etoh use, missed medication, stress)

EKG can be a quick initial test to pick up electrolyte abnormalities causing seizure.

Seizure meds

*Medications for seizure management

Elise comment: In the Peds ED there is frequently the availability to get an EEG rapidly for a child with a new onset seizure.  This rapid turn around of an EEG can be very helpful to neurology to make good management decisions for the patient.

Case 1.  Etoh withdrawl seizures can be treated with Ativan first line and Phenobarbitol second line for persistant  seizures despite Ativan.    Christine comment: You can push the upper dosage range of Ativan in the ETOH withdrawal patient. Their GABA receptors are down regulated.   Elise comment: If your Ativan is not working in any type of patient, start thinking about underlying causes such as hypoglycemia, hyponatremia, INH poisoning, intracranial hemorrhage.

20mg/kg loading dose works for Phenytoin, fosphenytoin, Phenobarbital, Valproic Acid, and Keppra.  Basically 20mg/kg works for all commonly used epileptics other than Ativan.

Check the patient’s tongue for lacerations and hematoma.  Serious tongue injuries  could pose an airway obstruction issue.

Neuro exam in infants

Look for:

Balled fists with thumbs tucked in, this is an upper motor neuron sign (Babinski sign in infants)

Altered/increased tone

Abnormal Reflexes

Eye deviation

Elise comment: Midazolam IM is a very rapid acting anti-seizure medication that is useful in kids.   Eye deviation is an important finding in kids to identify a seizure.

30-50% of patients with pseudo seizures also have epileptic seizures.  This makes diagnosis and management possible psuedoseizures very difficult. 

Carlson                      Antidotes

antidote-chart-forcpcs2008-1-728.jpg
antidote-chart-forcpcs2008-2-728.jpg

*Antidotes

Isopropyl alcohol ingestion is not an indication for fomepizole.  It is metabolized to acetone.

Lead has 3 potential antidotes: BAL,  EDTA, and Succimer.   Peanut allergy is an absolute contraindication to BAL therapy.

Gyromitra mushrooms look like a brain and can cause seizures.  The antidote is pyridoxine (also the INH antidote/INH also causes refractory seizures).

gyromitra mushroom

*Gyromitra mushroom

Sodium Bicarbonate is an antidote for TCA poisoning.  The sodium overcomes Na Channel blockade.  The Bicarbonate decreases free TCA.   Andrea prefers bolus dosing with 1-2 ampules while closely observing the patient and the monitor.

Calcium channel overdose can cause bradycardia, hypotension, and hyperglycemia.  The antidote is the “perfusion salad”: calcium chloride, glucagon, intralipid, atropine, and insulin euglycemic therapy.

Deadly nightshade contains bella donna toxin.  Bella donna can be thought of as a Benadryl overdose.  Both cause an anticholinergic toxdrome.  The antidote for anticholinergic toxidromes is physostigmine.   Physostigmine should not be used in undifferentiated coma or in mixed overdoses.  However, it is safe in pure anticholinergic toxidromes.  

deadly nightshade berries

*Deadly nightshade berries

deadly nightshade leave

*Deadly nightshade leaves  (note the little “ears” small leaves around the big leaf)

Paint stripper that contains methylene chloride if ingested will be metabolized to CO.  Methylene chloride gets metabolized by liver to CO and can cause CO toxicity.    Antidote is hyperbaric oxygen. 

Statistically the most common cause of CO toxicity in the summer is indoor grilling or using a grill on a screened-in porch. 

Guidelines For Hyperbaric Oxygen Therapy In Carbon Monoxide Poisoning Emergency Medicine Practice.JPG

*Hyperbaric indications for CO Toxicity

Causes of cardiac glycoside poisoning: digoxin, oleander, foxglove, Lilly of the valley, and the practice of toad licking.   Antidote is FAB fragments

West      Fever in Kids

Rectal temperature measurement is most accurate.   The temperature that determines a fever is 38C.

fever algorithm under 3 months

*Fever algorithm for kids </= to 3 months

For febrile kids (with immunizations up to date) over 3 months of age with no source on physical exam the next best diagnostic study is to check the urine.

Brain damage is not a complication of fever due to infection.

Faculty comment: Parents frequently under-dose Tylenol and Ibuprofen.  Check the dosing parents are giving to kids if the antipyretic is not bringing down the fever.

Harwood comment:  You may not need to do an LP on febrile kids age 30-60 days old if they have the following:

Well appearing child

Normal urine

Normal ANC

Normal Procalcitonin

This decision rule has not been validated. 

Elise comment: Right now there is a grey zone (based on age of infant) of whether there is a need to do an LP.  There is reasonable practice variation in non-toxic appearing, febrile kids age 30 days to 60 days.   Most faculty have a cut-off at 6 weeks, but up to 8 weeks is reasonable.  Ill appearing kids at any age require LP.

fever management table

Fever Algorithm for ages Newborn to 36months

Bonder     Patient Satisfaction

Patient satisfaction rates are a factor in medicare reimbursement.

Research has shown that patient satisfaction measurements are biased and don’t correlate with clinical quality metrics.   Also, mortality rates tend to be higher in hospitals with high patient satisfaction levels.

Safety net hospitals and teaching hospitals regularly perform more poorly on patient satisfaction measurements.   There are geographic variabilities as well with California and East Coast hospitals usually scoring lower than other regions.

4 habit model

*Kaiser 4 Habits Model from Kaiser Health

Elise comment: Patient satisfaction measurements are not based on your clinical care or diagnostic accuracy.  They are based on how you communicate with the patient.

Christine comment: At the completion of the ED visit, take the time to explain to the patient what the tests showed and how they ruled out dangerous diagnoses.

Patient satisfaction measures

*Patient Satisfaction Measures

Conference Notes 7-15-2015

If you don't see images, please scroll to the bottom and click "Read in Browser"

Burns     EKG Basics

 

*EKG Intervals

Harwood comment: when determining the QRS interval, measure at the lead with the widest QRS.

General Approach to Evaluating an EKG

  1. Heart Rate
  2. QRS: wide or narrow?
  3. Rhythm: Regular or Irregular?
  4. P waves
  5. Axis   If the QRS is predominantly positive in I and II or AVF it is a normal axis
  6. ST changes
  7. LVH, QT interval
  8. Special signs (osborne wave, brugada sign, WPW)

 

*Rule to determine Axis

 

*Simplified LVH Criteria

 

Ianitelli     Procedural Sedation

 

*Comparison of Common Sedation Agents

 

*Comparison of Succinylcholine with Rocuronium

 

Case 1.   19yo male with GSW to abdomen.  Patient is hypotensive and tachycardic.  Barounis comment: I would use low dose etomidate to avoid hypotension.  Give a higher dose of succinylcholine due to poor perfusion of muscles in shock.   Elise comment: I would use ketamine to avoid hypotension.   Harwood comment: I would use reduced dose of etomidate.

 

Case 2.   32 yo female with altered mental status.   Hypotensive, tachycardic, and febrile.   O2 sat is 92% on RA.  

Harwood and Girzadas comment: Delayed Sequence Intubation with ketamine. Pre-oxygenate then give succinylcholine to intubate.   ED Pharmacist comment: Have push dose pressors ready for any hypotension that would develop.

 

Case 3.   72 yo female found down. Patient has right hemiplegia.  Patient has been on the floor for at least 18 hours.   BP is 100/60,  Pulse of 70.

 Consensus was using ketamine or low dose etomidate to avoid hypotension.  Consensus was using rocuronium to avoid hypokalemia in a person with possible rhabdomyolysis/hyperkalemia/renal failure.    Samir Patel comment:  Succinylcholine is safe in chronic renal failure patients but be prepared to manage any hyperkalemia.  In the setting of acute renal failure, rhabdomyolysis, or hyperkalemia, avoid succinylcholine.

 

*Contraindications to Succinylcholine

 

Case 4.   58 yo male with a Gluteal Abscess.  Normal vital signs.   Most people felt ketafol would be the optimal choice.  Playing devil’s advocate, Kelly Williamson felt that using pain control and light sedation with fentanyl and versed could also work.  The benefit of this approach would be to avoid procedural sedation and any complication of sedation.

 

Kennedy    Use of Pressors

 

*Pressor Chart

 

Norepinepherine is your workhorse pressor.  It is probably the first choice for most non-hemorrhagic shock situations.

 

Septic shock:  first choice norepinephrine, second line is epinephrine or vasopressin

Anaphylactic shock: First choice is epinephrine,  second line is vasopressin

Spinal Shock plus bradycardia: Epinephrine,  second line is dopamine

Spinal Shock with normal rate:  First choice is norepinephrine

Cardiogenic shock:  First choice:  Use a combination of dobutamine for inotropy with norepinephrine to reach a MAP of 65. 

Pediatric Septic Shock (cold extremities)  First choice is epinephrine,  second line is dopamine

Pediatric Septic Shock (warm extremities)  First choice is norepinephrine

Hypoplastic left heart syndrome with shock: First choice: Milrinone   Second line: Dobutamine

 

Push dose pressors:  Give during RSI in a hypotensive patient.   Can also use in other clinical situation with expected rapid/short duration episode of hypotension

Epinepherine 1ml of cardiac epinephrine  added to 9ml of saline in a syringe.  You can give 1 ml of this diluted epinepherine every 2 minutes as needed for hypotension.

 

Myers     Ventilator Management 

 

Ventilator management in the lung injured patient who doesn’t have asthma or COPD:   Start on assist control.  Keep volume low, tidal volume of 6 ml/kg of ideal body weight.  Use the ventilator rate to provide appropriate ventilation.  Don’t change tidal volume to adjust ventilation.  15 breaths per minute is a reasonable starting point.  Start FIO2 at 100% and rapidly titrate down to FIO2 of 40% if possible to hit goal of O2 saturation >90%.  You can also use PEEP to improve oxygenation.  PEEP stents open alveoli to improve oxygenation.  PEEP can negatively impact the patient’s preload resulting in hypotension.   Keep plateau pressure less than 30.  Plateau pressure is the measure of risk of barotrauma.  It is a measure of the alveolar barostress.

 

*Suggested FIO2/PEEP Table   

 

*ARDSnet Protocol

 

Obstructive Strategy  (Asthma, COPD Ventilator Management):

Lower the patient’s  respiratory rate to a rate of 10 or even 8. Increase expiratory time by setting the I:E ratio to 1:5.  Permissive hypercapnea is OK.  Keep ph >7.2  Set PEEP to 0.

 

*Obstructive Ventilator Strategy

 

In the crashing intubated asthmatic patient, your first move is to disconnect the ventilator.  Then compress the chest manually to improve exhalation of air. 

 

Htet    Introduction to Codes

 

Have the same approach every time to every Code you manage.

Start with IV, O2, Monitor, Vitals, Dexi, EKG,

Get help from nurses and techs, respiratory therapist, and other physicians.

Proceed in your management based on the data you gather from these first two steps.

 

Case 1.  64 yo female who is unresponsive following an mvc.

 Start with IV, O2, Monitor, Vitals, Dexi, EKG,

Get help from nurses and techs, respiratory therapist, and other physicians.

EKG shows TCA toxicity. 

Consider intubation to potect airway

Give IV fluids

Consider pressors if hypotensive.

Give IV bicarbonate to narrow the QRS, and improve cardiac output and blood pressure.

 

*TCA EKG  Note tall, wide terminal R wave in AVR and Wide QRS diffusely

 

Case 2.  65 yo female with fever and cough, hypotension and tachycardia.

Suspicion is septic shock. 

Start with IV, O2, Monitor, Vitals, Dexi, EKG,

Get help from nurses and techs, respiratory therapist, and other physicians.

Intubate the patient considering the adjuncts of push dose pressors or delayed sequence intubation.

Give IV fluids, start antibiotics, give pressors if patient’s BP is not volume responsive.

Get appropriate labs including blood cultures and lactate.

 

Harwood comment: Bipap has been shown to decrease intubations but data demonstrating that it decreases mortality is difficult to find.

 

Case 3. 80 you female with sudden right hemiplegia. 

Start with IV, O2, Monitor, Vitals, Dexi, EKG,

Get help from nurses and techs, respiratory therapist, and other physicians.

Get rapid CT of head

Identify last known normal neurologic function

Perform NIH stroke scale

Give TPA if indicated

 

Samad    Intro to Renal Ultrasonography

 

*Anatomy of Kidney on U/S 

 

*Chronic Kidney Disease usually demonstrates smaller than normal kidneys, and decreased sonographic differentiation between cortex and medulla.

 

*Ureteral Pelvic Junction kidney stone

 

*Mild Hydronephrosis   The key to identify mild hydronephrosis is seeing hypoechoic areas within the normally hyperechoic renal sinus.

 

CT is better than U/S for evaluating for pyelonephritis and it’s complications.

 

The first three differential diagnoses for enlarged kidney are:  DM, multiple myeloma, and HIV nephropathy.  The fourth is an infiltrative process like amyloidosis, lymphoma, or leukemia.

 

There was a discussion of a ureteral jet seen on ultrasound.  A ureteral jet is a sudden rapid flow of urine from the ureter.  If the ureteral jet is not seen, it suggests a significant ureteral obstruction.  Elise comment: If you have a ureteral jet on one side and not the other it suggests a complete obstruction of the side without the ureteral jet.  The finding of a unilateral absent ureteral jet may be a useful marker when trying to convince consultants to place a percutaneous nephrostomy in a patient with a suspect obstructed/infected kidney. 

 

 

*Ureteral Jets on U/S using the Doppler U/S

 

Levine     Understanding the Body’s Response to Traumatic Experiences

 

Sometimes the great care we strive to give patients in the ED can result in psychological trauma to those same patients despite our best efforts.

 

Psychological stress/trauma can result in real physiologic responses (fight or flight response)

 

Co-Regulation speaks to how caregivers can help a person experiencing emotional or psychological trauma cope with that event.   We can control our tone, pace, and volume of our voice to calm and reassure patient.  Gentle, appropriate touch and pleasant facial expressions can also calm and reassure the patient.

 

Physician strategies to diminish psychological trauma to the patient:

Allow family/friends to be with the patient

Educate patients about their diagnosis and what to expect both in the ED and when they go home.

Call Patients by their name and speak to them, not about them.

Remain calm and make a conscious effort to monitor your tone, volume, and rhythm of voice.

Use light touch on the arm, hand or shoulder  to calm the patient.

Use verbal therapy.  The following are a few examples:  We are going to take care of you.  We are going to make you feel better.  You can say:  I am here for you.  We are going to get you through this.   Life is really hard.  You have been through some really tough stuff.   Validate what patients are feeling and be careful to not undermine their thoughts, fears, or concerns.  

 

Bonaguro     Sepsis Update

 

 

Sepsis continuum

 

When you suspect sepsis tell the nurse or SIMs that the patient is a Code Sepsis

Get an initial lactate and a 6-hour lactate

Septic Shock requires 30ml/kg crystalloid bolus.  Give an initial 2 liter bolus instead of a 1 liter bolus.  If you are worried about causing CHF give 2 liters in 500ml aliquots and frequently re-assess the patient.

Do a 6 hour re-exam including vital signs, cardiopulmonary exam, cap refill exam, pulse evaluation and skin exam.   OR  Do a bedside cardiac ultrasound and passive leg raise test.  OR Place a central line and get a CVP.

You must write your sepsis H&P note within 6 hours of the patient’s ED arrival.

An EMR fix for these new requirements is the “ED Sepsis Advisor Power Plan”

This is a more advanced power plan in FirstNet than the ED Sepsis Power Plan.   The Sepsis Advisor Power Plan has most of the above testing and documentation requirements built in.  

 

 

 

 

 

 

Conference Notes 7-8-2015

If you don’t see the images, please scroll to the bottom and click on

“view in browser”.

Lambert   Ultrasound Physics Intro

 Low frequency sound waves travel further than high frequency sound waves.  

Low frequency probes have less image quality than high frequency probes but they can visualize much deeper structures.

High frequency probes have low penetration but better picture quality in the more superficial tissue.

 

Near Field is the image area on the screen closest to the probe.  Far Field is the image area on the screen further away from the probe.

In all probes, the near field images are usually better than the far field images.  There is more sonic information coming back to the probe from the near field than the far field.  

 

Image Orientation:  The probe indicator should correspond with the indicator on the screen.  In general for all imaging except echocardiography, the probe indicator should be pointing either to the patient’s head or the patient’s right side.  When performing echo’s, the probe indicator is oriented to the patient’s left side. 

 

 

*Shadowing is a high attenuating artifact.  Shadowing from a gallstone.

 

 

*Low attenuation  structure results in an enhancing artifact (more echogenic distal to the structure).  There is also a refraction artifact in this image causing an edge shadow.

 Set your image depth so that the structure you want to image takes up 7/8 of the screen.   If the image of the organ of interest  takes up less screen space than that you are wasting the real estate of your screen.

Set your gain so that the image echogenicity is consistent from top to bottom of the screen.

Always scan a structure in two planes.   Identify the boundaries of the structure you are interested in.

 

Lambert    7UP  Scan

 

Mike credited Chris Kerwin with inventing the term 7UP scan.

The 7UP scan is a rapid ultrasound evaluation for the patient in shock.

It is a similar protocol to the RUSH (Rapid Ultrasound in Shock) Protocol.  

Editors note: It was easier for me to grab some images of the RUSH protcol off the internet then get 7UP images.  My apologies to Mike and Chris.

 

 

*RUSH Protocol

 

 

*RUSH   Algorithm

 

The 7UP  protocol also images the RUQ,  Subcostal area, Abdomen, Pelvis, then Chest.

 

In a hypotensive patient with abdominal pain, fluid in Morrison’s pouch and a positive UCG there is nearly a 100% likelihood of ruptured ectopic pregnancy.

 

*Cardiac Tamponade with RV compression.  Note the scalloped/compressed appearance of the RV.

 

*Type A Aortic Dissection

 

*Ultrasound consistent with PE.  Typical RV is triangular in shape and smaller than LV.  PE with increased right side pressures will show a proportionately larger RV.

 

*IVC in Hypovolemia

 

*IVC in Hypervolemia

 

*AAA with Thrombus

 

When looking for pneumothorax, image 3 sites on each hemithorax.  If you don’t see normal sliding of the pleura, a pneumothorax is present.

 

 

*Ultrasound Imaging of Pneumothorax.  The left image is normal lung that shows pleural sliding and hyper-echoic comet tails.  The right side image is a pneumothorax where pleural sliding and comet tails are absent. 

 

Lambert/Frazer/Chastain/Chan and other Team Ultrasound Members             Ultrasound Lab

 

 

 

 

 

 

 

 

 

Conference Notes 7-1-2015

BG:  ph=7.0, PO2=200, PCO2=30, HCO3=5.   FIO2=100%  O2 on ven

If you don't see images scroll to the bottom and click on "read in browser"

Jeziorkowski/Girzadas  Oral Boards

 

Case 1.  55yo male presents in coma.  The patient is found to have an anion gap metabolic acidosis and an osmolal gap.  Urine shows calcium oxalate crystals and EKG has prolonged QT interval.  Patient was diagnosed with ethylene glycol toxicity.  Patient was intubated for airway protection.   Fomepizole was given to block alcohol dehydrogenase.   Nephrology was consulted for emergent dialysis.  Optimal care included a bicarb drip and thiamine and pyridoxine co-factors given to optimize metabolism of glyoxylic acid.   

n  Anion Gap = Na-(CL + HCO3) and should be 15 or less

n  This Case: 140-(105 + 5)=30

 

 CAT MUDPILES mnemonic for anion gap acidosis

 Cyanide, CO, CHF

 AKA, Aminoglycosides

 Theophylline, Toluene, Tylenol

 Methanol

 Uremia

 DKA

 Paraldehyde, propylene glycol

 Iron, Isoniazid, Inborn Errors

 Lactic acidosis

 Ethylene glycol, Ethanol (lactate)

 Salicylate, Starvation Ketosis

 

Case 2.  7 yo male shot in right thigh during a drive-by shooting.  Patient was resuscitated with IV fluids 20ml/kg  and IV PRBC transfusion of  20ml/kg.   Patient did not have hard signs of vascular injury.   Patient did have abnormal ABI of right lower extremity.   CTA of right thigh showed vascular injury.  Patient went to OR. 

 

Hard Signs of Vascular Injury mandate operative repair

Pulsatile bleeding

Pulsatile hematoma

Bruit/thrill

Absent pulse distal to injury

Ischemic signs (pain, pallor, coolness, paralysis)

 

EAST Guidelines

Hard signs should prompt surgery without angiogram

ABI <0.9 or other soft signs should get  CT angiography

CT angiography is the primary diagnostic study

No Hard/soft signs and normal ABI can be discharged

 

Case 3. 4 yo male fell in pool and was submerged for 1-2 minutes.  When rescued, patient coughed and started crying.  There was no loss of consciousness and no required resuscitative efforts.  On arrival to the ED , the patient had normal mental status,  normal vital signs, clear lungs, pulse ox =97% on room air and CXR was normal.  Patient was observed in ED for 4-6 hours and remained asymptomatic. Patient was discharged home. 

  • A patient that was immersed in a liquid and suffered any sort of respiratory event is considered to have suffered a drowning
  • Asymptomatic patients with clear lungs, normal pulse ox, and normal CXR can be discharged after 4-6 hours of observation.
  • Criteria for Admission(altered mental status, abnormal vitals,  P/O<95, respiratory symptoms, abnormal lung sounds, abnormal CXR)

 

 

Htet    M&M

 

There are studies to document that morbidity and mortality in teaching hospitals increases during July and August.  It is a real phenomenon.

 

Case: 65 yo male presents with hematuria.  He is also hypoxic and hypotensive.   Patient is on warfarin for atrial fibrillation.

Labs showed an INR of 7 and a lactate of 5.   WBC= 13.6.  HGB=8.  CXR showed a RLL infiltrate and effusion and a possible small right side apical pneumothorax.

 

FEIBA and vitamin K was given.  Patient was also treated for pneumonia/sepsis with Ceftriaxone and Azithromycin.

 

Later that day, patient dropped his pressure and became hypoxic.  He deteriorated further into a PEA arrest.   A chest tube on the right was placed during resuscitation out of concern for possible worsening pneumothorax and pus came out in the tube.  

Diagnosis was empyema and warfarin coagulopathy.

 

Dr. Htet’s Teaching Points:

  1. Intubate early in course of illness
  2. Consider CT of Chest to further evaluate the lung findings of pneumonia/effusion.  
  3. Consider a chest tube early if empyema is in the differential diagnosis.
  4. Pursue the cause of shock aggressively.  Elise comment: An ABG can be useful to help determine the cause of shock.   The Emergency Physician should set for themselves time limits to get the patient out of shock.  If the time limit is not met more aggressive measures need to be started.
  5. Harwood comment: I agree the ABG is an underused tool for figuring out the cause of sepsis.  Rechecking the HGB is important.   If the HGB is falling below 8 in a patient with and elevated troponin, transfuse.
  6. Harwood comment: If you have a sick patient with an infiltrate/pleural effusion and on the same side the patient has a small apical pneumo (air in the hemithorax) you need to consider empyema.

 

Bedside ultrasound is a good way to identify if patient has an empyema.

 

 

*U/S of empyema

 

Empyema needs to be drained. Think of it as an abscess in the chest, or similar to an obstructed/infected kidney.   All these pus collections need emergent drainage.

 

4 reasons to intubate

  1. Protect airway
  2. Failure to oxygenate
  3. Failure to ventilate
  4. Ease the work of breathing

 

Bottom line if you see a patient with pneumonia and pleural effusion do a CT or bedside ultrasound to differentiate empyema from para-pneumonic effusion.  If you identify empyema,  thoracentesis or chest tube to drain the empyema is indicated. 

 

 

Burt    Acid/Base 4 Step Method of Analyzing an ABG

 

Step 1.   Identify abnormal values on the ABG

 

Step 2. Calculate the anion gap

Na - (Bicarb + chloride)= Anion Gap

 

Step 3. Apply the Rule of 15

Bicarb +15 should =  the PCO2 and Last two digits of the pH.   This step will identify a “hidden respiratory” process.    As an example if the PCO2 is higher than expected by the rule, there is a respiratory acidosis in addition to the primary metabolic acidosis.  If the PCO2 is lower than predicted by the rule, there is a respiratory alkalosis.

ABG:  ph=7.1, PO2=200, PCO2=30, HCO3=10.   FIO2=100%  O2 on vent

Bicarb  +15 = 25.  Based on the rule of 15 the pH should be 7.25 and the PCO2 should be 25.  However the measured pH is 7.1 and the PCO2 is 30.  Because the PCO2 is higher than predicted by the rule there has to be a respiratory acidosis in addition to the metabolic acidosis. 

 

Step 4.  Calculate the Delta Gap

Subtract  the delta gap (AG – 15) from the normal expected bicarb (24) to identify the predicted bicarb. 

If  actual measured bicarb is higher than predicted then there is a hidden metabolic alkalosis.   If the actual measured bicarb is lower than predicted then there is a hidden metabolic acidosis.

Example:  Anion Gap =25,  Hco3 = 15.    25-15=10.    24-10=14.  The actual measured bicarb (15) is pretty close to the predicted bicarb (14) so there is no “hidden” metabolic process.

Example 2: Anion Gap= 30, HCO3= 20    30 -15 = 15 .   24 -15 =9    The predicted bicarb  (9) is less than the measured bicarb 30 indicating a “hidden” metabolic alkalosis in addition to the metabolic acidosis.

 

Elise comment: The delta Gap calculations (Step 4) are difficult and rarely will make a clinical difference in the ED. You probably can get by with the first 3 steps alone. 

 

VBG’s and ABG’s have reasonably similar values for ph, pco2, and bicarb.  The PO2 is the value that is significantly different between an ABG and VBG.

 

*ABG  vs VBG

 

 

Parker     ACS

 

 

*STEMI vs NSTEMI

 

 

*Posterior EKG

 

 

* Antero- Lateral STEMI

 

 

*Inferior STEMI

 

*RV Infarct (III with more ST elevation than II.  Also ST elevation in V1)

 

Inferior STEMI’s tend to have bradycardia and blocks.  They can have papillary muscle rupture.   Overall inferior STEMI’s tend to do well.

 

Anterior STEMI’s  tend to develop CHF, shock, wall rupture, and malignant arrhythmias.   Anterior STEMI’s have higher mortality than inferior STEMI’s

 

Right ventricular MI’s are very volume dependent and can develop hypotension.  Be very cautious with IV NTG.  Give fluid boluses to support pressure.

 

Treatment of STEMI’s :   ASA,  Nitroglycerin,  Heparin are the basics. 

 

 

*Sgarbossa Criteria for diagnosing AMI with LBBB or Paced EKG

 

Myers       Administrative Lecture

 

EM Doctors and nurses should meet and update the patient together prior to discharge or admission.

 

Be careful when ordering cervical/vaginal diagnostic studies.   Frequently the wrong swabs are sent.  Be careful to label your specimens. On the Label, note the time you actually obtained the sample.  Samples need to be sent to lab within one hour of being obtained.  If the test was ordered initially but then later you decided not to gather samples, cancel the order. When obtaining GC/Chlamydia samples use the blue swab only.  The current CDC recommendations state you can swab anywhere in the vaginal vault to obtain GC/Clamydia samples.  You don’t have to swab the cervix.  Give the patient the time frame of 7-10 days for GC/Chlamydia results.

 

If you are swabbing an extra-genital site for GC/Clamydia you have to use an E-swab for GC culture and UTM  (universal transport media/influenza medium) for chlamydia culture.  You don’t use the GC/Chlamydia swabs we use for vaginal/urethral testing on extra-genital sites.

 

Medication reconciliation is critical to patient safety.  A good resource for patients’ medications is in Firstnet.   Under Orders, click on “external medication references.”  When the window opens up, you may have to click on refresh multiple times to get the list.  You can also click on “medication list” under Orders in Firstnet.  Use other resources as well: patient, family, nursing home records, prior records, and primary physician.  

Elise and Kelly comment: It is the physician’s responsibility to identify the medications the patient is on that could be possibly causing the acute condition.  It’s not the physician’s responsibility to put the list into the computer, but in the end we are responsible for identifying critical meds like warfarin or rivaroxaban.

 

 

 

Conference Notes 6-3-2015

If you don't see images scroll to the bottom an click on "view in browser"

Gore    M&M

Elderly female presents from the nursing home with leg wounds.  Patient was in the hallway and generally not that sick appearing.   However technically, she had SIRS criteria based on heart rate of 95, RR=22, and temp of 38.1 .   Labs showed a lactate of 7.3.   Patient had bands on her CBC as well as toxic granulocytes.  The patient’s  appearance and location in the hallway belied the fact that she was actually in severe sepsis.

 

*SIRS Criteria 

 

Surgery consultants did not feel that the patient’s leg wounds required surgical debridement.

 

Sayger comment: General surgery lately has been pushing back on cases outside of the abdomen.  Other conference attendees noted an impression that there is sometimes disagreement by general and specialty surgical services  about which service has primary responsiblity for some body areas.  Elise comment: These disagreements need to go up the food chain to involve the Departmental Chairs and Administration.

 

The patient developed hypotension.  Fluid resuscitation resulted in decreased O2 sat and bilateral rales in lungs.  It was later learned that patient had a history of diastolic heart failure.   At this point sign out occurred. 

The sign out team placed a central line for pressor support.

Patient was admitted to ICU.  Blood cultures grew out strep.   Consultants generally agreed that the wounds were chronic and did not require debridement or amputation.

Christine comment: The leg wounds may have been a portal of entry for strep bacteremia/sepsis but they may have not been infected per se.

 

Faculty discussion about the cognitive biases we face with ED patients, especially hallway patients.  Humans have two cognitive systems at work in their brains.  System 1 is fast, intuitive, and relatively effortless. That is the system we depend on the majority of the time.  System 2 is analytical but slow, lazy, and effort intensive. It lags behind system 1.   As ER docs we have to make sure our system 1 (intuitive) is built on good knowledge of decision rules like the HEART score or PERC rule.  Then we neeed to take some time with our cases and seek out other cognitive cues to give our system 2 time to do it’s job to analyze the clinical situations we face.   To read more on this topic I would suggest the book, “Thinking Fast and Thinking Slow” by Daniel Kahneman

 

*Thinking Fast and Thinking Slow

 

 

Knight   Bites and Antibiotics

 

5% of untreated dog bites and 80% of untreated cat bites will get infected.   Treat with Augmentin for both if the patient is not penicillin allergic.  Elise comment:  Augmentin for bite wounds is known as “Dogmentin”

For pen allergic patients you can give clindamycin/floroquinalone dog and human bites or clindamycin/doxycycline for cat bites.  For pen allergic kids you can give Zithromax.

 

High risk wounds: All cat bites, deep dog puncture wounds, hand wounds, and immunocompromised patients, and any injury going to the OR are considered high risk.

 

Cat scratch disease results in regional lymphadenopathy 7-12 days after a cat scratch.  It is due to Bartonella infection. Antibiotics are not indicated unless adenopathy is painful or the patient is immunocompromised.  If you treat, 5 days of azithromycin is reasonable.   Elise comment: Kitten scratches are more prone to cat scratch disease than cat scratches.

 

Treat all human bite wounds as infected.   Fight bite wounds are high risk.  You should give at least one dose of IV antibiotics, check an xray, splint the injured joints, and consult hand surgery for these cases.   Human bite wounds are polymicrobial infections and may also have eikenella.  Augmentin is again a good choice for these wounds.  For pen allergic patients you can use clindamycine and a flouraquinalone.

 

*Fight Bite

 

Bites from rats, mice, and squirrels are also treated with Augmentin. 

Rat bite fever: Rigors, fever, polyarthralgia.  Mortality 10%.  Treat with Augmentin/Unasyn.

Livestock bites have more potential for tissue damage and systemic infection. 

 

Rabies is a risk for bites from dogs, bats, monkeys, skunks, raccoons, and foxes.  Rabies is not a risk for rats, mice, squirrels.

When giving HRIG, infiltrate around the wound and give the remaining HRIG IM at a site away from the vaccine site. 

HDCVaccine is given at days 0,3,7,and 14.

Elise comment: Small animals like rats don’t carry rabies because they have a high metabolism and die before they can infect someone.  However, groundhogs are an example of a lagamorph/rodent  that can cause rabies.

Cirone comment: There was a case of rabies in cattle in Illinois in 2011. 

Elise comment: Bats are the most common rabid animal in Illinois

Faculty consensus: A patient bit by your average family dog in cook county does not need rabies prophylaxis.  If the dog has been exhibiting aggressive behavior or unusual behavior or it was one of a pack of feral dogs or the dog just was transported from out of the country  then give rabies prophylaxis.

 

It is recommend to suture  the following bite wounds: Face or scalp, less than 6 hours old, simple with no underlying injury, and non-immunocompromised patient.   

 

Parker      COPD Exacerbations

 

COPD Exacerbation=Acute change in baseline dyspnea, cough, or sputum production.

 

Be cautious about O2 administration.  Aim for an O2 sat of @92%.  Higher O2 saturations can diminish the patient’s respiratory drive and result in hypercapnea. In COPD, supplemental O2 should raise the O2 sat relatively easily.  If you need a lot of O2 to raise the O2 sat start thinking about alternative diagnoses like PE or pneumoia.

 

Give antibiotics for admitted patients, increased sputum purulence, increased dyspnea, and bipap’d or ventilated patients.  Basically this will include most patients and definitely any patient that is a little bit sicker.  In general you can use a macrolide, flouroquinolone, or doxycycline.  For severely ill patients be sure to cover pseudomonas with Levoquin, Cipro, or anti-pseudomonal cephalosporins. 

 

Magnesium is cheap and safe.  Give 2gm IV in severe cases.  A recent study for asthma (not COPD) showed no benefit of magnesium in asthma so extrapolating to COPD suggests that it may not be of benefit.   Motzny comment:  Magnesium may have some respiratory parameter benefits (disease oriented end points) but no patient oriented outcome benefits have been identified (no improved mortality, lowered intubation, shorter stay, etc)

 

Bipap has been shown to decrease intubations by 65% in COPD patients.

When intubating, bag with a low rate to allow time to for the patient to expire trapped air.  The best ventilation strategy would a be low rate (10 or les),  I:E ratio of 1:5, assist control,  tidal volume of 6 ml/kg, 100% initial FIO2, and peep of 5 or less.  Keep plateau pressures <30.  Following the peak pressure is not clinically helpful.   Ventilate with permissive hypercapnea. 

 

You can use manual decompression (take the patient off the ventilator and manually compress chest to force air out) on an hourly rate to increase air movement out of the chest.

 

Harwood/Girzadas comment: Use a lung protective strategy with TV of 6ml/kg, Rate of 8-10, and start with FIO2 of 100% and titrate down to an O2 sat of about 92%.   Braden comment: If you keep the O2 sat higher you may benefit by knocking out the patient’s respiratory drive and allow the vent to more completely control the ventilations. 

 

High flow, High humidity nasal cannula can deliver 40L/min of O2 with added humidity.  It opens up alveoli and provides positive airway pressure.   Girzadas comment: Will high FIO2 drive up the O2 sat and decrease respiratory drive?  Elise and Bradon response:  You can set the flow rate, humidity rate, and FIO2 so you can use this modality and set the FIO2 at 40% or something other than 80-100% FIO2 and not knock out the patient’s respiratory drive.   Febbo comment: This will be an important modality for hypoxic lung problems like pneumonia maybe more so than COPD.

Barounis comment:  High flow O2 gives the patient some peep (3-5). It stents open the airway.   You should use this when intubating a hypoxic patient.   Elise comment: When is bipap a benefit over high flow O2?  Barounis response: CHF is probably the time to use bipap.   It does provide more peep than high flow O2.   Purely hypoxic patients like PE’s and pneumonia’s will be better served by initially trying high flow O2 than bipap.

 

Barounis recommendations:

Bipap for CHF and COPD

Hi Flow O2 for hypoxic pneumonia or PE.  Use for pre-intubation.  Also consider trying this modality initially for severe asthma. 

For the patient who is struggling with the bipap mask, you can also try high flow O2.

 

Kennedy   5 Slide FU

41yo NH patient with dyspnea and tachypnea.   Patient is febrile and tachycardic.  Patient’s limbs are diffusely rigid.   CBC with 26.7 WBC’s.  Lactate is 5.1.   Initial CK was 253.  CSF was clear with 2 wbc’s.  UA showed wbc’s but no bacteria. 

 

Most likely diagnosis was neuroleptic malignant syndrome vs. sepsis. 

 

*Neuroleptic malignant syndrome

 

*Neuroleptic malignant syndrome DDX

 

Patient’s CPK increased markedly.  Neuroleptic malignant syndrome was diagnosed.

 

Treatment for neuroleptic malignant syndrome: Correct hyperthermia, IV fluids, give benzo’s and dantrolene for muscle relaxation. 

Andrea comment: If there are no serotonin meds on board, give bromocriptine as well.  Bromocriptine can worsen serotonin syndrome so avoid this medication in any cases where serotonin syndrome is a possibility (serotonergic meds on board). 

 

Toerne      Acute ETOH Withdrawl

 

Case 1.  63 yo male with hx of hypertension.  He drank a pint of vodka daily.  Presented with tachycardia and hypertension. Patient had a facial contusion and a left hand injury.  Magnesium was low and LFT’s were mildly elevated. Head CT was normal.

The DSM5 uses the term Alcohol Use Disorder as a broad term that encompasses alcohol abuse and alcoholism.

Chronic ETOH use results in an extrinsic tonic sedative effect on the brain.  The brain compensates by increasing excitatory activity.  This compensatory response includes glutamate release and up regulation of NMDA receptors.  GABA receptors are down-regulated and there is decreased sensitivity to benzo’s.   Then when the compensated patient ceases ETOH use, you get a brain that is hyper-stimulated. 

 

4 ETOH Withdrawal Syndromes

 

  1. Seizures:  Brief tonic clonic seizures early in the withdrawal state.  <3%develop status epilepticus.  40% will go on to DT’s.
  2. Uncomplicated Alcohol Withdrawal: 6-24 hours after cessation of drinking. They have tremor, tachycardia, diaphoresis but no altered mental status.
  3. Hallucinosis: Ted believes this diagnosis does not actually exist. Hallucinations in this syndrome are not associated with disorientation.  Very few cases of this has not been reported.
  4. Delerium Tremens: Starts 2-4 days after cessation of alcohol use.  The earlier DT’s occur following alcohol cessation portends worsening symptoms/signs.  The hallmark of DT’s is disorientation, global confusion, and hallucinations.   As with other forms of delirium, DT’s encephalopthy will wax and wane.   The patient also will have peripheral effects of a hyperadrenergic state.   Be sure to rule out other causes of delirium.  You can have DT’s without prior seizures.  Seizures and DT’s are separate entities.  They can go together or not.

 

There was a group discussion regarding the example of a patient who normally lives at the 500-600 range of serum ETOH levels.  When that person drops to the 250 range they develops DT’s.   There was consensus that “relative ETOH deprivation” can result in alcohol withdrawal syndromes even with relatively high serum ETOH levels.

 

For Ted the key question to ask a patient is “How many days per week do you drink?”  Then “When you drink how much do you drink?”  Patients will minimize their alcohol use.  One way to get at how much ETOH a patient drinks is to ask how much alcohol the patient buys. 

Clinical cues to occult alcohol abuse: swollen hands and feet, hypomagnesemia, mild LFT elevation, mild macrocytic anemia, mild thrombocytopenia.

 

Criteria for admission: Significant medical diagnosis, significant trauma diagnosis, and severe uncontrolled withdrawal symptoms. Seizure may or may not indicate need for admission.

 

Management: Provide a quiet environment,  frequent re-assessments of the patient, evaluate for other medical problems. 

Give medications to get the patient to a point where they look like they are sleeping. 

 

High dose lorazepam >10-15mg/hr can result in lactic acidosis due to the propylene glycol in lorazepam.  Lower dosing regimens don’t cause lactic acidosis.

Febbo comment: To get rapid control of a patient having severe withdrawal symptoms, IV valium may be a better choice than lorazepam due to it’s rapid peak action at 5 minutes.  Ted agreed with this.

 

Phenobarbital acts synergistically with lorazepam.  It is cheap and long acting.  The combination of phenobarbital and lorazepam can increase the risk of respiratory depression and hypotension.

Ketamine is an option because it is an NMDA receptor agonist.   Ted feels this is a great choice for rapidly sedating a very agitated ETOH withdrawal patient.  He gives 0.5 mg/kg as the initial dose.

 

Dexmedetomidine has not shown a shortened ICU stay despite resulting in overall lower benzo doses.

 

Ted’s approach/New ED Protocol for Alcohol Withdrawl:

Lorazepam  2mg, 4mg, 8mg, escalating Q 10 minutes.  If not controlled,

Lorazepam 5mg/hr continuous infusion.  If not controlled,

Phenobarbital 10mg/kg IVPB over 30 minutes.  If not controlled,

Start a phenobarbital continuous infusion.  If not controlled,

Start a ketamine infusion.

 

Elise and Harwood were uncomfortable with starting at 2mg dosing of Lorazepam.  They wanted the first dose of lorazepam to be 4 mg.

 

Katiyar        EM Billing and Coding   Increasing Procedure RVU’s

 

Be sure to write a procedure note for any procedure, even removing a crayon from a kid’s note.   Document whether you used sedation for the procedure.  Sedation for procedures increases the RVU’s for that procedure.

For burns, in addition to your H&P, document the procedure of cleansing or debridement, as well as applying antibiotic ointment and a dressing.  You can bill for any burn management procedures if you do it.

 

Document cardiac monitoring before and after treatment.  This increases your RVU’s.  Note the rate and rhythm.  Document the rate and rhythm before and after any intervention.

 

Document your assessment of the pulse ox value (adequate oxygenation, hypoxia).  Document if any intervention you took based on that assessment.

 

Document post splint care. This increases RVU’s.   You have to write something like: Neurovascular status evaluated after splint applied. Patient has normal pulse and capillary refill.  Patient has normal movement/sensation of fingers.  Splint is comfortable for patient and not overly tight.

 

Be careful with voice dictation. There tends to be a lot of errors.  You have to review your dictation for errors.  Dictation errors can be embarrassing and problematic if your chart is reviewed for medico legal reasons.

 

Length and location of the wound determines the reimbursement.  Measure each wound with the tape measure that is wrapped around the 4X4 gauze in the suture kit.  Don’t guesstimate the laceration length.  Guesstimation can under or over-estimate the length.   Digital and regional blocks also increase RVU’s.  Document them in your procedure note if you do one.

 

Optimize your diagnoses because it improves your RVU’s and improves the hospital case mix index.  For example, Acute STEMI demonstrates severity of illness better than Acute Chest Pain.  DKA is better than hyperglycemia.  Hospital reimbursement improves with a higher case mix index.

 

We can bill for smoking cessation counseling if you discuss with the patient and or family for more than 3 minutes.  You need to document your conversation and the length of time of the conversation in the chart.

Conference Notes 5-27-2015

 

Permar      STEMI Conference

Case 1.  Sorry I missed this first case

 

Case 2.  41 yo male with no prior cardiovascular diagnoses presents with chest pain and diaphoresis.  In the ED the patient was hypotensive.  EKG showed an inferior/posterior MI.   A Code STEMI was called immediately.

 

*Inferior-Posterior MI

 

Patient developed torsades in the ED.  He was resuscitated multiple times.  He made it to the cath lab.    Cath showed severe multi-vessel disease.  Despite a balloon pump, ICD, medical resuscitation, and angioplasty the patient died.  

 

Dr. Dia comments: Using a larger volume balloon in the balloon pump (50ml vs 40 ml) is more effective.   There are also temporary ventricular assist devices or ECMO that could be useful is these situations.  Balloon pump by itself has not been shown to improve 30 day mortality.

Dr. Levin and Silverman comments: There are Impella devices that could be possibly placed in the cath lab.  These are percutaneously placed left ventricular assist devices.

Dia comment: To support a cardiac arrest patient with ECMO it has to be VA ECMO so that you are supporting the left heart not just the right heart.

Harwood comment: This patient’s anatomy was fatal.  Be very careful with vasodilators (nitro) in patients with inferior/posterior MI’s. Think RV infarct.  Also intubate these very sick patients prior to sending them to the cath lab. The cardiologists agreed with these comments.

Silverman comment: I have developed a lower threshold to intubate patients prior to cath.  If we have to intubate during the cath, it slows down and interrupts the cath procedure.

Levin comment: Intubation in the cath lab takes more time than in the ED. Patient selection for these unstable patients is critical for making the best decision whether to go to cath or not .

Group discussion regarding taking post-arrest patients to cath lab. There is an EM push to get post-arrest patients to the cath lab.  Cardiology has the some contrary concerns that post-arrest patients frequently die in the cath lab or shortly therafter.  Careful patient selection and good communication between EM and Cardiology is crucial.

 

Case 3. 55yo male had out of hospital V-Fib Arrest.  Patient was resuscitated pre-hospital.  ED EKG showed an anterior MI. 

  

*Anterior MI

 

Silverman comment: This patient was continuously unstable in the ED with repeated V-Fib arrests.  He should go to the cath lab, but he was never stabilized long enough to make cardiac cath possible.  He recommended loading the patient with amiodarone and if you can get a period of time with a blood pressure present, get him to the cath lab.

 

Be sure you document you cath lab decision making.  Document that the patient is unstable, that the case is complex.  This will take them out of the 90 minute timing issues.

 

This patient eventually stabilized for cath lab. He had a 100% LAD occlusion. He was placed on balloon pump.   Patient was transferred to MICU and had another V-fib arrest.  He was resuscitated but then had PEA and died.

Dia comment:  PEA suggests to me either a respiratory problem or ventricular rupture.  Silverman comment: He had respiratory failure due to pump failure and worsening pulmonary edema.  We are moving toward having an ECMO device or Impella device to support these patients beyond a balloon pump. Cooling the post-arrest patient is also super-important. We can place a cooling catheter in the cath lab if necessary.

Harwood comment: Cooling is not indicated for patients who are conscious after arrest.

 

Case 4.  64 yo Patient presents with chest pain and subtle EKG suggesting posterior mi.  Dia comment that with AVR showing subtle elevation and diffuse mild ST depression also consider LAD lesion.  Harwood noted prolonged qt interval as well.

Posterior EKG showed very subtle st elevation in V7-9.  There was no consensus in the audience about calling the STEMI based on these EKG’s.

Patient was managed medically.  He went to cath 1-2 days later. Cath showed severe multi-vessel disease with LAD and marginal branch with 90% occlusions.

Silverman comment: This patient had very elevated blood pressure so there is a lot of room to medically manage this patient with nitrates and beta blockers and cool them down without emergent cath.

 

Cardiology comment: In the initial EKG the anterior t waves are inverted suggesting anterior ischemia rather than posterior mi which may have st depression with upright t waves.

 

*Anterior ischemia

 

*Posterior MI

 

Kadar/Putmann     Oral Boards

 

Case 1.  26 yo female with abdominal pain for 5 days.  Patient has fever and tachycardia.  History reveals vaginal discharge.  Only medication is an oral contraceptive.   Physical exam demonstrates lower abdominal tenderness and CMT and vaginal discharge.  Diagnosis is PID with a TOA.  Critical actions:  Get a pregnancy test,  provide pain control, obtain cervical and vaginal specimens, get a pelvic ultrasound, start IV antibiotics (IV cefoxitin and IV doxycycline plus IV flagyl), Consult GYN.

 

Case 2.  2 year old child with fever and rash. Heart rate/pulse/respirations are normal.  Dexi is 100. Rectal temp is 39.  History reveals 5-6 days of fever and now has a diffuse erythematous rash.  Review of systems is negative.  Exam shows red cracked lips and bilat conjunctivitis.  Patient has bilat hand swelling. There is cervical lymphadenopathy.  Skin shows a diffuse erythematous rash.  Diagnosis is Kawasaki’s disease.  Critical actions are Starting ASA and IVIG. Order ECHO and consult ID or rheumatology.   Rash can be very nonspecific. 20% of untreated patients will develop coronary artery aneurysms 2-6 weeks after illness.  These aneurysms can lead to sudden cardiac death later in life.

 

*Kawasaki’s

 

Case 3.  3 year old with possible nasal FB. Vitals are normal.  Airway is patent, no respiratory distress or choking.  Mom states the child said he put something in his nose and has pain. Exam demonstrates FB in the left nostril.  Critical actions: Remove FB with kissing technique (mom blows air into child’s mouth rapidly with non-affected nostril occluded) or forceps or balloon catheter technique.

 

McVicker  (312-550-1941 You can text Tricia with any safety event)  

Root Cause Analysis

 

No one goes into healthcare to hurt patients or break rules.  If an error happens it is because our system broke down.

 

Root Cause Analysis is a structured process for identifying the causal or contributing factors underlying adverse events or other critical incidents.

 

Accidents in health care almost never stem from a single cause. They come from a mix of failures or work conditions that align precisely to slip through every existing defense.

 

 

*Swiss Cheese Model of Error

 

The goal of RCA’s is to learn from adverse events and prevent them from happening in the future.  We talk about what happened and why it happened.  Then we try to come up with a plan to prevent it from happening again.

 

Residents are very helpful to the RCA process because they know the micro processes of patient care.

 

ACMC does RCA’s when there is an event of unexpected death or severe harm to a patient.  RCA makes safety a real priority and focuses on how the system failed.  It does not focus on personal blame.

 

The RCA process is protected under the Illinois Review Act.  It is not discoverable for malpractice cases.

 

The RCA process identifies the probable causes of an event, looks for reasons why they happened, and then identifies ways to prevent such an event from happening again.

During the RCA a timeline of events is developed.  Then causal events (the why’s) are mapped out in a flow chart.

 

Cash  M&M

 

“Failure is instructive”  “We do not learn from experience…we learn from reflecting on our experiences.”

 

Case 1. 35 yo female with hyperglycemia and LLQ pain.  Ct showed ureteral stone.  UA showed blood and WBC’s. 18 hours later, patient was febrile and tachycardic.  Creatinine had bumped to 2.58.  Patient became hypotensive.  IV antibiotics were started.  Nephrostomy tube was placed.    Patient later expired from sepsis.

 

Reflections: Take ownership of sign outs. They are truly your patients.  Don’t slip into autopilot mode.  Make sure that planned interventions are getting done.

Girzadas comment: Keep a lookout for these obstructed kidneys with any signs of infection (fever, wbc’s in the urine).  There is urgency in getting these kidneys drained by IR with a nephrostomy tube.

 

Case 2. 30 yo female had left flank pain.  UA shows leukocytes and blood. Ultrasound shows left renal hydronephrosis.   Vitals show HR=108 and BP =106/72.  Patient was discharged.    2 hours later the patient returns to the ER unresponsive.  Vitals were 39.8C and hypotensive/tachycardic.   At this point lactate is 7.  IV Fluids, IV antibiotics were given and patient was intubated.  An emergent nephrostomy tube was placed.  Urine showed klebsiella.  Patient improved and was discharged home.

 

Reflections: Pay attention to vital signs.  Beware of communication failures between nurses and docs.  Be vigilant with hallway patients and on busy shifts.

Harwood comment: The receiving sign out team has fresher brains and should be willing to re-evaluate the patient later during their shift.

Natalie Htet comment: Using ultrasound can show hydronephrosis in these patients but I have seen two patients who had some hydro seen on ultrasound come back to the ED and large ureteral stones were then identified on CT.  Hydro on ultrasound does not give you much data on the size of stone or severity of obstruction.

 

Case 3.  65 yo female with abdominal pain and fever.  Tachycardia and hypotension.  Patient notes some dark urine.  On exam patient has suprapubic tenderness and mild right CVA tenderness.  Urine shows blood and leuks.  Diagnosis is pyelonephritis.  A Ct was performed.  Patient had an obstructing stone in right ureter. Right nephrostomy drainage tube was placed.  Urine culture shows Klebsiella. 

 

There was a discussion concerning whether we should be doing ultrasounds or CT on pyelonephritis more frequently to rule out ureteral stone/obstructed kidney.  Mila Felder suggested we review a series of cases to develop a more robust clinical pathway for pyelonephritis.

 

Sola comment: I have been burned on signouts that say  “This patient is being discharged so we don’t need to talk about this patient”  A patient planned for discharge started vomiting and needed further care and I knew nothing about the patient.  Harwood comment: I even ask about the patients that died on the last shift.  In case family later shows up.

 

Ede     5 Slide Follow Up

 

27 yo female with RLQ abdominal pain and syncope.   UCG was positive.  Pain radiated to right shoulder.  Exam shows BP 107/74   HR=95.  Minimal blood in vaginal vault. Cervix closed.  UA shows large blood.  HGB=11.6.   Beta HCG= 67.  U/S showed free fluid in RUQ.   Patient was taken to the OR.  A left tubal pregnancy was removed.   The patient had a ruptured ectopic pregnancy with a Beta HCG of 67. 

 

Risk factors for ectopic pregnancy: PID prior tubal surgery, IUD, assisted reproduction, and previous ectopic.

 

Triad of ectopic pregnancy: Abdominal pain, vaginal bleeding, and amenorrhea.

 

Findings suggestive of ectopic on U/S: free fluid, mass, no intrauterine pregnancy.

Beware of heterotopic pregnancy in patients who are undergoing assisted reproduction therapy.

Harwood comment: Ectopic pregnancies are reported/known to have low Beta HCG’s.  So don’t let a low Beta HCG lull you into complacency.

 

Munoz    5 Slide F/U

 

4 yo female with abdominal pain and lethargy.  Patient had a history of Graves disease and is on propranolol.  Child had been sleeping all day.   Vitals show tachycardia and tachypnea. No fever.  Child was lethargic on exam.  Stool and emesis were heme positive.  Labs significant for anion gap acidosis with an osmolal gap.  ED docs considered toxic alcohol ingestion. Patient was admitted to PICU and eventually was diagnosed with thyroid storm.  7 days later the patient had thyroidectomy.

 

*Burch-Wartofsky Criteria

 

*Treatment for Thyroid Storm

 

Harwood comment: This seems like a case of apathetic thyroid storm in a child.  This could be a case report material.     

 

 

 

 

 

 

 

 

 

Conference Notes 5-20-2015

If you don't see images scroll to the bottom and click "view in browser"

Ryan      Musculoskeletal Emergencies   Study Guide

 

Spinal stenosis clues: patient walks in flexed forward position.  Patient has lower extremitypain when walking.  Patient has pain relief with sitting.  No pain with sitting is the most specific historical clue.  Spinal stenosis causes pain with exertion similar to vascular claudication.  However, spinal stenosis pain does not resolve when the patient stops walking and remains standing.  Vascular claudication should resolve when the patient stops walking and remains standing. Spinal stenosis pain won’t resolve until the patient can flex his hips by sitting , leaning forward, or squatting.

 

*Spinal stenosis

 

*Normal healthy spine

 

Adequate muscle relaxation is the most important factor in successful shoulder reduction

 

 

*Testing for supraspinatus Empty Beer Can Test (Jobe’s Test) Pain and weakness resisting downward pressure on the arm suggests a tear in the Supraspinatus Tendon

 

*Jobe's Test

Harwood comment: For knee injuries that swell immediately, there probably is a hemarthrosis.  That is the only mechanism for a knee to swell immediately.  For knees that swell up overnight that is more likely a reactive effusion.

 

Cauda Equina syndrome is associated with urinary retention.

 

*Cauda equine syndrome

 

*U/S of urinary retention

 

Reimplantation of amputated digits is basically all about the thumb.  Discusss with the hand surgeon prior to discarding any tissue.  Keep the amputated finger wrapped in saline soaked gauze.  Place the saline gauze-wrapped digit in a plastic bag and place the bag on ice. Keep the digit in a cooler that stays with the patient.

 

 

*Gout vs Septic Arthritis

 

Jeziorkowski     Trauma to Cervical Spine

 

*Nexus C-spine Rule

 

*Canadian C-Spine Rule

 

Elise feels the Canadian Cspine rule is superior to the Nexus rule based on it’s increased specificity. If the Canadian C-Spine rule indicates imaging, go to CT rather than plain films.

We also discussed the practice of placing an axial load on the patient’s head to check for tenderness.  It was felt among the group that if you do the Canadian Cspine rule first with head turning prior to axial loading, it probably isn’t a dangerous practice to axial load, but it also is not useful.  Axial loading is probably not a useful test for assessing the Cspine.

EAST recommends CT as the imaging method of choice for the Cspine.  Plain films are only indicated for a young, non-obese, non-extremely muscular person with a low-risk mechanism of injury.

 

Chris discussed multiple unstable C-spine fractures

 

 

*Unstable Cspine Fractures

 

*Cspine Fractures

 

*Flexion Teardrop Fracture

 

*Hangman’s Fracture

 

 

If CT Cspine is negative and patient has persistent severe pain or paresthesias you need to get an MRI.  There is an entity called SCIWORA, which is a spinal cord injury without radiographic abnormality.   This injury is usually a ligamentous tear.  This injury can be best picked up by MRI.  There was consensus among the attendings that flex-ex films are inferior to MRI for picking up ligamentous injury.  Also all the attendings felt that patients who have some persistent midline posterior neck pain following a negative CT cspine but have low to moderate-risk mechanism and no neuro symptoms can go home without an MRI.  It should be said that EAST still recommnends MRI for patients with persistent pain following negative CT. Most attendings felt it was unnecessary to MRI all these patients.  A selective approach based on severity of pain, mechanism, and neuro symptoms seems reasonable.

 

EAST Guidelines recommend removal of c-collar after a negative CT scan of Cspine in a comatose patient. This is a brand new recommendation.  There was some collective head scratching on this recommendation by the faculty. Most faculty felt they would still MRI this type of patient (head injured, comatose, in c-collar following negative c-spine CT)

 

Cash/Lovell      Clinical Teaching

 

 

*One Minute Preceptor.   Getting a learner to commit to a diagnosis they feel is most likely and defend that commitment is a key to this teaching method.

 

SPIT  teaching method:  Ask the learner to give a differential diagnosis in each of the following SPIT categories

Serious

Probable

Interesting

Treatable

This is an easy way to teach on almost every patient.

 

 

Create a motivating environment.  Be an encouraging and supportive teacher.  Give the learner specific feedback.

 

 

3 Step approach to teaching procedures

  1. Assess your learner’s skill level
  2. Teach Concepts.  This includes repetition of the tasks of the procedure.  Introducing a small bit of variation regarding how to perform the procedure helps with performance and long term retention of the skill.  With more advanced learners’, focus on key portions of the procedure and discuss how to refine their technique.  “What if” simulation scenarios a la Harwood asking you how would handle this procedure if the patient was pregnant or was a pediatric patient or had septic shock can improve the advanced student’s learning.
  3. Directed Feedback.  Be specific in your analysis and suggestions to the learner.

 

 

*Deliberate Practice

 

 

*Peyton’s 4 Step Model

 

*Elise then showed a video of how to make rabbit ears based on Peyton’s 4 Step Model.  This skill was clearly mastered by multiple residents

 

Jamieson/Watts    Oral Boards

 

Case 1.  Middle aged man presents with syncope while watching the Hawks game.  Patient has history of cold symptoms for 2 days. Physical exam is unremarkable except for URI findings.  EKG shows brugada changes. 

 

*Brugada EKG V1-3 has RBBB pattern with ST elevation

 

*Brugada types

 

Critical actions: Consult cardiology and admit for AICD placement.

 

Case 2.  16 yo female with hematemesis and syncope.  Patient is tachycardic.  Patient has history of autoimmune hepatitis.  Patient develops massive hematemesis in the ED and becomes unresponsive.  Critical actions: Patient is intubated to protect airway. 2 large bore IV’s, IV fluid resuscitation, IV PRBC transfusion. Protonix and Octreotide.   Consult GI and admit to ICU.  Ideal management includes NG tube and IV antibiotics (Rocephin or a Quinalone).

 

(International Journal of Hepatology 2011) Currently, it is recommended that short-term antibiotic prophylaxis, a measure that reduces bacterial infections [23], variceal rebleeding, and death [24], be used in every patient with cirrhosis admitted with gastrointestinal hemorrhage [20, 25, 26]. Different antibiotics have been used in different trials compared with placebo (Table 1, [27–30]). Bacterial infection is commonly associated with variceal hemorrhage and appears to be an independent risk factor for failure to control bleeding [31] and predicts both early rebleeding and death [32, 33]. The routine use of prophylactic broad-spectrum antibiotics has shown a marked improvement in outcome in acute variceal hemorrhage. Routine intravenous ceftriaxone or postendoscopic norfloxacin reduces rebleeding rates compared to on-demand antibiotics (Table 2) [24, 29, 34–36]. A Cochrane meta-analysis of antibiotic prophylaxis in cirrhotic patients with gastrointestinal bleeding involving 12 trials with 1241 patients evaluated antibiotic prophylaxis compared with placebo or no antibiotic prophylaxis. Antibiotic prophylaxis compared with no intervention or placebo was associated with beneficial effects on mortality (RR 0.79, 95% CI 0.63 to 0.98), mortality from bacterial infections (RR 0.43, 95% CI 0.19 to 0.97), bacterial infections (RR 0.36, 95% CI 0.27 to 0.49). They concluded that prophylactic antibiotic use in patients with cirrhosis and upper gastrointestinal bleeding significantly reduced bacterial infections, and seems to have reduced all-cause mortality, bacterial infection mortality, rebleeding events, and hospitalisation length. These benefits were observed independently of the type of antibiotic used [37, 38].

 

Case 3. 26 yo female with right index finger pain. Temp is 37.9C. Other vitals are normal.  Patient has history of gardening and jabbed her right index finger on something sharp in the soil and suffered a laceration.   Her tetanus status is up to date.  Right hand demonstrates Knavel’s signs of flexor tenosynovitis on exam. Testing in the ED shows a positive pregnancy test.  Patient also has a penicillin allergy. Critical actions: IV Clindamycin (Pregnancy Class B) to cover Staph, x-rays of right hand, Hand surgery consult for surgery.

 

*Knavel’s Signs of Flexor Tenosynovitis

 

Harwood comments:  Great job telling the examiner what you know and thinking out loud.  Get a rhythm strip on the first patient ASAP don’t wait for the EKG.   You don’t need a CT scan of the head as part of a routine syncope work up.

 

Faculty felt that the tenosynovitis patient needed broad coverage including gram positive and gram negative coverage.  Elise and Harwood felt a third generation cephalosporin plus/minus clindamycin would be reasonable.  Girzadas felt clindamycin to cover MRSA was indicated.

This will give the alumni a laugh, but Girzadas advised doing a rectal exam on all syncope patients.

 

 

Lee   5 Slide Follow Up

 

35 yo female 28 weeks pregnant presents with abdominal pain. Patient was hospitalized at an outside hospital a week earlier for renal stones and ureteral stent placement.  Vitals were normal/afebrile.  On exam the patient had right lower quadrant and right flank tenderness.  CBC showed a WBC of 15.  Urine was negative for infection and positive for moderate blood.  The initial thought was that the patient was having recurrent ureteral colic. Renal ultrasound here at ACMC showed mild hydronephrosis but no stones. 

 

A pelvic ultrasound performed here at ACMC showed a right adnexal mass c/w right ovarian torsion.  Laparotomy removed a necrosed dermoid tumor.  It is conjectured that calcifications in the dermoid tumor that was adjacent to the kidney may possibly have complicated the ultrasound of the kidney performed at the outside hospital. This could have led to the initial diagnosis of nephrolithiasis and renal colic.  We however did not have those images to review.  

 

Pregnancy is associated with an increased risk of ovarian torsion.  Torsion is most likely to occur between 10-17 weeks gestation.  Torsion presents with pelvic pain, adnexal mass and nausea/vomiting.

The ovary moves upward toward the kidney as pregnancy progresses.  This may make the diagnosis more difficult.

DDX for RLQ pain in a pregnant patient =ruptured cyst, ovarian torsion, appendicitis, tubo-varian abscess.

 

*MRI demonstrating right ovarian torsion with concurrent pregnancy.  Notice how high that right ovary is.

 

 

Bamman   5 Slide Follow UP

 

3 yo male presents with fever and drooling  Fully immunized.  Child was cranky with chapped lips.  There were no intra-oral ulcerations.  The patient did not improve in the ED with magic mouthwash. And would not take a popsicle.  Because this patient looked sicker than the usual kid with stomatitis and he did not have intra-oral ulcerations, a lateral soft tissue neck film was ordered.

Lateral soft tissue neck film showed possible epiglottitis.

 

Patient was taken to the OR for intubation.  On laryngoscopy patient had epiglottitis.  There was some tracheal pus. Tracheal cultures showed staph aureus.  Patient was treated with Vancomycin and Zosyn and decadron.  Patient recovered well after 3 days of intubation.  Staph areus is becoming more common as an etiology of epiglottitis.

 

*Epiglotitis.  This patient has a thumb sign and a vallecula sign.  The vallecula sign is when the air column in the vallecula does not extend to the hyoid.

 

 

 

 

 

 

 

 

 

 

 

 

 

 

 

 

 

 

 

 

 

Conference Notes 5-13-2015

If you don't see images, scroll to the bottom and click "view in browser"

Burns/Collins    Oral Boards

 

Case 1. 27yo female passed out at home.  HR=62, BP=79/51, Afebrile.   Patient notes abdominal pain.   No PMH.  Patient is taking OCP’s only intermittently and UCG is positive.  Patient is persistently hypotensive despite IV fluid resuscitation.   FAST exam shows free fluid in Morrison’s pouch.  Diagnosis was ruptured ectopic pregnancy.  Critical actions were IV Fluid and Blood resuscitation, Rapidly getting the patient to the OR, and Giving rhogam  (pt was RH negative).

 

*Free Fluid in Morrison’s Pouch.  This finding with shock and a positive pregnancy test has a positive predictive value of >90% for ruptured ectopic pregnancy.

 

Case 2. 27 day old child with diarrhea, fever, and “not acting right.”  T=100.3, HR=160, BP=75/50  RR=46  PulseOx=96% on RA.  Decreased urine output.  Exam shows signs of dehydration.   Patient was given 20ml/kg bolus.  A septic work up including LP was begun.   LP showed 200 WBC’s.  Treatment for meningitis was begun.    Critical actions:  IV fluids, septic work-up, always consider evaluate for possible abuse, treat with antibiotics for meningitis.

 

Case 3. 38yo man brought in by his father.  Father is concerned that patient is suicidal.  Vitals are normal.   Patient refused to speak with ED physician.  Patient’s dad said that patient took an overdose of tylenol and threatened to shoot himself.   Tylenol level was 350.  Patient was treated with NAC per protocol.  Patient’s duffel bag was taken away from him.  When searched, his bag contained a handgun.  As a note to the readers, the conversation between Dr. Burns and Dr. Collins (AKA Angry Jerry) was a classic!  Dr. Burns calm, flat responses to Dr. Collins angry outbursts rivaled a comedy skit.  Critical actions: Prevent patient from leaving ED, make sure patient’s possessions are taken away from him to prevent him from using a weapon in the ED.  Obtain an acetaminophen level.  Treat with NAC.  Consult Psychiatry and Poison Control.  Admit to ICU.

 

*Rumack Matthews Nomogram

 

The approved 20 hour IV dosing regime is complicated and is performed as follows:

Administer an initial loading dose of 150 mg/kg IV over 15 to 60 minutes (we recommend 60 minutes).

Next, administer a 4 hour infusion at 12.5 mg/kg per hour IV (ie, total of 50 mg/kg over 4 hours).

Finally, administer a 16 hour infusion at 6.25 mg/kg per hour IV (ie, total of 100 mg/kg over 16 hours).

 

72 hour oral protocol — The 72 hour oral (PO) dosing protocol for N-acetylcysteine treatment has been used successfully in the United States for more than 30 years, and consists of the following:

A loading dose of 140 mg/kg PO, followed by

A dose of 70 mg/kg PO every four hours for a total of 17 doses

*NAC Treatment Protocols from Up to Date

 

Girzadas comments: Positive FAST exam in the setting of shock and positive pregnancy test has a positive predictive value of over 90% for ectopic pregnancy.  You absolutely have to LP febrile kids under one month of age.  The physical exam of kids this age is unreliable for identifying a toxic or sick/meningitic neonate.  You have to speak with family and or friends of psychiatric patients.  The psychiatric patient’s history is extremely unreliable and can mislead you to underestimate their suicidality.  You have to take the time to corroborate the patient’s history with family or friends.

There was a discussion about whether a neonate should get steroids for meningitis.  According to Up to Date: Dexamethasone is not indicated in the treatment of bacterial meningitis in infants younger than six weeks or in those with congenital or acquired abnormalities of the central nervous system.

 

Katiyar    Toxicology of Plants

 

Basic Classification of Plants

 

Alkaloids contain nitrogen.   They have bitter taste.   Most of them end in the three letters “ine”   Strychnine, Ephedrine, Morphine.

 

Glycosides  end in “in”   Digoxin, Salicin (metabolized to asprin)

 

Triterpenes     Tetracanabinol,   urushiol (poison ivy)

 

Proteins/peptides/lectins:  Ricin,  mushrooms (extremely poisonous)

 

Phenols: Capsaicin, St. John’s Wart

 

Case: 15 month has crying and drooling after chewing on a plant

 

Elephant ear plants contain calcium oxylate.  This can cause drooling, swelling of tongue and mouth.  Eye irritation.

 

Poinsettia leafs have a latex sap that can irritate mouth and throat and cause vomiting.   Poinsettias are not fatal, there have been no reported fatal cases.

 

Rhubarb leaves can cause nephrotoxicity.  Don’t eat the leaves, they can damage your kidneys and cause hypocalcemia.  

 

Jimson weed seeds if ingested can cause a significant anticholinergic toxidrome.  Jimsom weed is ubiquitous along roads.

 

*Jimson weed

 

*Salvia divinorum    Ingested leaves can cause hallucinations, coma, and memory loss.

 

Nutmeg is also hallucinogenic.   Morning glory seeds are hallucinogenic.

 

Oleander, fox glove, and lily of the valley can cause digoxin toxicity.  Patients with toxicity from these plants may need very large doses of  Digibind.  The lab testing of digoxin levels due to toxic plant ingestion can be falsely low.

 

Rhododendrons can cause hypotension, cardiac arrhythmias, and death.

 

 

*Water hemlock can cause seizures and there are case reports of death.  It looks a lot like Queen Ann’s Lace.

 

*Poison Ivy/Oak/Sumac.   These plants contain urushiol.

 

Don’t eat green tomatoes or green potatoes.  These can cause GI irritation and hallucinations.

 

Ackee fruit can cause elevated ammonia level with CNS alteration and GI distress.

 

Colchicine is contained in glory lilly and can cause multi-organ failure.

 

Ricin is extracted from the castor bean.  Jequirity pea is similar to ricin but 30 times stronger.   Both can cause hemorrhagic gastroenteritis and multi-organ failure.

 

Apple seeds in large amounts can be fatal due to cyanide toxicity.

 

Apple seeds and castor beans are not toxic if swallowed whole.

 

Bottom line to this lecture:  Don’t eat plants that aren’t sold in the vegetable section of your grocery store.   If you eat plants from other places bad things can happen.

 

Regan     Five Slide Follow Up

 

23 yo male fell while drinking and using heroin 2 days prior to ED visit .  Patient presented to the ED with left side weakness.  Patient also had left side facial swelling and left arm and leg swelling.

Lab testing showed dark urine and a serum CK of  58, 000.  Neuro work up was basically negative.  It turned out that patient had been laying on the floor for a prolonged period of time.  This resulted in rhabdomyolysis. 

 

Rhabomyolysis is diagnosed with CK level 5X normal.  Large blood in urine is a clue to rhabdomyolysis that should be followed up with a measured CK level.  CK levels>5000 increases risk of renal injury.

Treatment is aggressive IV fluids.  Alkalinize urine with bicarb  (3 amps of bicarb in 1 Liter of  D5W) .   There is no clear evidence that bicarb drip has an advantage over IV saline.

 

Orthopedic Lab

 

We had 5 stations to work on various exam and reduction techniques.

 

ICEP and SAEM Conference Pictures

 

 SAEM SONOGAMES Team  Drs Hart, Htet, Ede.

ICEP  Lindsay Purnell

 

Jennifer Cash at ICEP

Rachel Kadar at ICEP

 


Conference Notes 5-6-2015

Some further Feedback from Alumni on Peritonsillar abscess management: In my practice, the process has changed and the ENTs admit most of these patients for IV antibiotics reporting that only a small percentage require drainage.  Just FYI we may want to see what the ENT literature says on this issue...

Kristin McCabe, MD

Sent from my iPhone

Urumov           Study Guide Ortho

 

Bohler’s Angle  <25 degrees suggests calcaneal fracture.  The calcaneus is the most commonly fractured tarsal bone.   If a patient has a calcaneus fracture, look for associated fractures like hip and lumbar spinal fractures.

 

*Bohler’s Angle

 

 

*Knee Exam Tests

 

 

*Pseudo-Jones’s Fracture (Avulsion Fracture)  Treatment is with ace wrap and cast shoe and patient can bear weight.

 

*Jones Fracture   Treatment is non-weight bearing, post mold and likely surgery.

 

 

Posterior knee dislocation pretty much mandates a CT Angio looking for popliteal artery injury.    Harwood comment: In 2015, there is no reason not to do a CT Angio for posterior knee dislocations.   Prompt diagnosis of a popliteal artery injury may save an amputation.   The only reasons to not do it would be a patient with a low GFR or allergy to contrast.

 

 

*Maisonneuve Fracture

 

 

*Maisonneuve Fracture

 

 

*Osteochondritis Dessicans    Occurs most commonly in adolescents at the medial distal femur.  It is not a traumatic disease.  It is a developmental disorder.  Treated with non-weight bearing.

 

 

*Thompson’s Test for Achilles tendon rupture.    You can also use ultrasound to diagnose Achilles tendon rupture.

 

 

*US of Achilles tendon rupture

 

 

*Lis Franc Fracture   Be alert for fractures at the base of the 2nd metatarsal and  imperfect alignment of the 2nd metatarsal with the middle cuneiform.  These two findings are markers for a Lis Franc fracture.

Balogun       M&M

 

59yo male presenting with generalized weakness.  Patient has chronic jaundice that has worsened.  He uses daily ETOH.   Vitals abnormal only for tachycardia of 117.  

Patient had some bruising on his chest.  Abdomen was distended but non-tender with no peritonitis.  Liver was enlarged. Stool was brown, hemoccult positive.   Patient was tremulous.

 

Patient was initially treated with CIWA protocol, IV fluids and vitamin supplementation.

Labs showed mildly elevated troponin, mild hypokalemia and anemia to a HGB of 7.2

Ammonia level was normal.   Coagulation studies were prolonged.

Imaging showed ascites and gallbladder sludge.

 

Patient was in the ER for 6 hours.  He remained hemodynamically stable.   When patient was evaluated by the admitting physicians in the ED, the patient was less alert.

A CT done at that point, showed an intracranial hemorrhage.

 

Considering the weak cirrhotic patient, do a broad work up.  Look for infection,  elevated ammonia level, GI bleeding, anemia, intracranial bleeding, and cerebral edema.

A CT head is recommended for cirrhotic patients presenting with weakness. 

Harwood comment: Is there data to support that recommendation?

 

Elise comment:  You have to be really careful with patients who have cirrhosis.  However I am not going to CT scan the head of every weak cirrhotic patient. I will scan them if their mental status is not normal or the mental status changes in the ED.

 

There was a lively discussion about the difficulties of evaluating a patient with chronic cirrhosis and weakness.  Most people felt that weakness alone would not prompt a CT head initially.  However, these patients need frequent re-evals in the ED to evaluate for any change in mental status.   At times re-evaluation is difficult due to the pressure in the ED to keep seeing the next patient.  You have to take the time to re-evaluate the patient if a nurse calls about the patient, if an admitting doctor or consultant raises a concern, or before signout.

 

Treatment for ICH in a cirrhotic patient: FFP is the recommended reversal agent in this setting. Vitamin K has questionable benefit but you should still give it 10mg IV.  Elise comment: Don’t give Vitamin K subQ in cirrhotic patients as it can cause hematomas.   Give it IV. Give platelets if the count is <10K.  FEIBA is not recommended.  Factor 7 is not indicated. 

Harwood and Elise comments:  If the cirrhotic patient has abnormal coagulation studies but no life threatening bleed, don’t try to fix the coagulation numbers.  These patients have complex coagulation abnormalities and trying to fix the numbers may actually make them hyper-coaguable.   

 

 

Kadar      Pain Management in Acutely Injured Patients

 

Risk factors for chronic pain: high pain intensity, long duration of pain, anxiety, depression, work safety, associated litigation, poor pre-trauma health status.

 

Opiophobia:  We sometimes avoid opiates due to concerns about respiratory depression, hemodynamic instability, masking neurologic injuries, and the subjectivity of pain.    Research shows that older adults tend to get treated less for pain, many trauma patients get no pain medication and there tends to be long delays to pain medication administration in trauma patients.

The research also has shown that opioid pain medication in trauma patients does not increase intubations or other complications.

 

 

*WHO Analgesic Ladder

 

Ketamine is an NMDA receptor blocker.  It has been used by anesthesiologist to treat pain and lower the need for opioid treatment. 

 

PCA analgesia administration in trauma patients has to be used judiciously so that you don’t hinder early mobilization of these patients.   Early mobilization is key to preventing chronic pain.

 

 

Regional anesthesia has the advantage of less sedation, less respiratory depression, and less constipation/nausea.

 

 

*Facial Nerve Blocks  For longer action of anesthesia use bupivacaine with epinephrine.  Note that all three  facial blocks line up with the pupil.   Harwood comment: To anesthetize the central forehead you have to place bilateral supraorbital nerve blocks.

Elise comment:   You can lay down a line of anesthetic parallel to the upper  eyebrow to anesthetize a broad area of the forehead.

 

 

*Local Anesthetics Dosing and Duration

 

 

 

*Wrist Block

 

 

*Wrist Block

 

 

Continuous epidural infusion of lidocaine or bupivacaine can be used for multiple rib fractures, thoracic or abdominal post-operative pain.

 

 

*Intercostal nerve blocks can also be used for rib fractures.   Insert the needle in the posterior axillary line. Direct the needle down to the rib initially and then re-direct below the rib to anesthetize the nerve. 

 

 West   Follow Up  Case

 

54 yo female not feeling well a week after total knee surgery.  Patient noted some decreased urination.

Vitals show hypotension 92/31  HR=90  O2Sat=85%

Incision site did not show signs of infection.

WBC=46 with toxic granulocytes. Creatinine=3.45.  Anion Gap=26

Urine shows blood and signs of infection.

CT chest shows pneumomediastinum, free intra-abdominal air.

IV fluids were given.  IV antibiotics were given.  Surgery was emergently consulted.

Initial thought was that the patient may have an esophageal injury due to intubation from her knee surgery.  Patient did go to surgery and was found to have a perforated diverticulum and some necrotic bowel. There was no esophageal injury.

 

Central obesity, smoking, and etoh use increase risk for diverticulitis.

Patients with diverticulitis should be referred for colonoscopy in 6 weeks to be evaluated for colon cancer.  10% of diverticulitis cases can’t be differentiated from colon cancer initially.

 

There was a lively discussion about this unusual case.  It’s still hard to see the final diagnosis.  Why would a perforated diverticulum have associated necrotic bowel.  Why the severe mediastinal air?  A lot of unanswered questions. 

 

Cirone   Follow Up Case

 

Patient presents with bloody diarrhea.  Just returned from Africa.

IDPH was called when the patient arrived to the ED and IDPH advised that the patient was unlikely to have Ebola.   Patient travelled from a country that is not Liberia, Guinea, or Sierra Leon.

ID consult agreed that the patient did not have ebola.   Stool cultures were sent. 

 

*Screening of Travelers for Ebola    Screening only occurs for travelers from Liberia, Guinea, and Sierra Leon.

Elise comment: This patient came from a country that did not have  reported ebola cases.  Be careful not to raise panic in the ED for a traveler not from Liberia, Guinea, or Sierra Leon.  

Christine comment: In the middle of the night, call epidemiology because they have the most up to date knowledge of the current hospital protocols.

 

 

 

 

 

 

Conference Notes 4-22-2015

Alumni Feedback from Conference Notes 2 weeks back:

 

Hey Dan, It's Valerie Merl again. Just want to give some input on peritonsilar abscess treatment in the community. Your note says - peritonsilar abscess : consult ENT to drain. This was the thought when I was in residency also. We don't have ENT at my hospital so I would transfer all of these patients to a hospital that has ENT. Until I found out that the ER doctors at that hospital were draining all of these abscesses and sending these patients home. ENT was only involved as a follow up. So I learned to drain peritonsilar abscesses. In many places this is a skill that is expected of an emergency medicine doctor.  

See, someone reads the emails you send out!

Thanks Valerie, really appreciate the feedback!   I welcome people’s insights on any of the topics covered in the conference notes.  If you send an email, I will put you in the conference notes as well. 

 

Bonder/Ryan        Oral Boards

 

Case 1.  50 yo  female presents with chronic back pain and acute dyspnea. CXR is normal.  ABG shows respiratory alkalosis and metabolic acidosis.    Salicylate level is elevated.  Follow up history identifies that patient  had been over-treating with ASA for her back pain.   Diagnosis is salicylate toxicity.   Critical actions:  Bicarb drip (3 amps of bicarb in a liter of D5 and run at 250ml/hr), IV fluids, trend ASA levels, and consult nephrology for possible dialysis.   Patients can get cerebral hypoglycemia from salicylate toxicity so keep serum glucose over 80.  Acetazolamide is contraindicated in salicylate toxicity.  Acetazolamide does alkalinize the urine but it also lowers arterial ph which promotes salicylate movement in to the brain.

 

Case 2.  55yo male presents with right shoulder pain. BP 170/90  P 115  R 20   T 37  P/O 99%.    Patient fell from tree and his arm is abducted at  the shoulder with the elbow pointed to the ceiling and the patient cannot move his shoulder.  Diagnosis is luxatio erecta.  Critical actions: Pain control, shoulder reduction.  Luxatio erecta has a high incidence of axillary nerve and artery injury.  However, there is not a recommendation that all these patients require a CT angiogram or ultrasound of the axillary artery after reduction.  It is acceptable to just palpate for good pulses proximally and distally in the injured extremity following reduction. If you have higher suspicion based on mechanism or exam then go on to imaging the artery.

 

*Luxatio Erecta

 

*Luxatio Erecta

 

*Luxatio Erecta Reduction

 

Case 3.  25yo female found by EMS personnel seizing in a park.  BP 165/100  P 130  T 37.3Blood sugar=100.  Narcan had no effect per EMS.  No known PMH.  Patient has another seizure in the ED.   IV Ativan stops the seizure.   Patient appears to be homeless.   Thorough physical exam reveals a gravid abdomen. Diagnosis is ecclampsia.    Critical actions: IV magnesium,  BP control,  monitor the fetus,  fetal ultrasound, possible delivery of fetus.

 

*Treatment for pre-ecclampsia/ecclampsia

 

Harwood comments:   Fentanyl dosing is microgram/kg.   For an adult  male with a shoulder dislocation he probably needs 75micrograms to 100 micrograms for a single dose.     For ecclampsia, in addition to giving magnesium, you also need to treat the blood pressure.

 

Elise comments: If you have a toxicology case on the boards, consult  poison control.  To optimize alkalinization when treating salicylate toxicity, check the urine ph not the blood ph.  Physiologically, the goal is to alkalinize the urine to increase urinary excretion of salicylate.

 

 

Berkelhammer       Ischemic Disease of the Gut

 

Congestive Hepatitis is when the liver gets swollen due to acute CHF.  This can also cause central zonal necrosis of the liver lobules.   AST can be specifically more elevated than ALT in the setting of shock liver/acute CHF liver

 

You can get hepatitis due to hypoxia and severe anemia as well.   Pregnant patients can get catastrophic antiphospholipid syndrome that results in liver infarctions.  This syndrome can be difficult to differentiate from HELLP syndrome.

 

Chronic mesenteric ischemia is called abdominal angina.  Patients will get pain shortly after eating.  Patients may loose weight due to not eating to avoid pain.   Smokers, diabetics and hypertensive persons are at higher risk for abdominal vascular disease.   Patients need to have double or triple vessel disease to get abdominal angina.  If you do a screening ultrasound or CT angio and the celiac and superior/inferior mesenteric arteries are patent, the patient does not have abdominal angina.   Treatment  for abdominal angina is angioplasty or endarterectomy.

 

Acute mesenteric ischemia has pain out of proportion to exam.  Labs can be initially normal but as time passes, the  WBC will go up, metabolic acidosis will develop, amylase and lipase will increase, and lactate will elevate.   If you see air in the portal circulation or in the bowel wall, that is a sign of dead gut. 

 

*Air in the bowel wall (pneumatosis intestinalis)

 

Think about mesenteric ischemia in patients presenting with abdominal pain in the setting of atrial fibrillation.

 

Aortic dissection can also cause mesenteric ischemia.  The dissection can occlude the celiac and superior/inferior mesenteric arteries.

 

Arterial mesenteric ischemia tends to give abrupt onset of pain.   Venous mesenteric ischemia can be insidious in onset.   Predisposing factors for venous thrombosis include portal hypertension, hypercoaguable states, and cancer.

 

Low cardiac output states can result in non-occlusive mesenteric ischemia.  The gut arteries can vasospasm in the setting of hypotension/low cardiac output.  

 

ED Treatment of acute mesenteric ischemia is heparin and vascular surgery consultation.

 

Ischemic colitis can present as a patient who wakes up in the middle of the night with acute onset abdominal pain and initially multiple brown bowel movements followed by diarrhea followed by bloody diarrhea.  Patients will have tenderness over the colon.  WBC can be elevated.   It can occur in any area of the colon.   The vessels perfusing the colon on the side opposite the omentum are prone to vasoconstriction and or occlusion.  Young people on vasoconstrictive medicines like sudafed can actually get ischemic colitis.

 

 

Ischemic colitis compared to small bowel mesenteric ischemia has less pain, gradual onset, colonic tenderness, bloody diarrhea, less sick.  Whereas small bowel mesenteric ischemia has sudden onset severe pain, no focal tenderness initially, no bloody diarrhea until late in the course. 

 

Celiac compression syndrome (median arcuate ligament syndrome)  The arcuate ligament can compress the celiac artery in thin patients.  They will have pain c/w chronic abdominal angina.

 

 

*Celiac Compression Syndrome

 

SMA syndrome  occurs in patients who have lost a large amount of weight (like 100 lbs).  The loss of omental fat squeezes the duodenum between the SMA and aorta.  The patient has basically a gastric outlet obstruction from this compression and they will vomit after eating.

 

*SMA syndrome

 

 

Remke      Management of Traumatic Arrest

 

Case 1.   25 yo male with multiple GSW’s to abdomen and chest.  Patient lost vitals in the field. 

 

What to set up prior to patient arrival:  Airway equipement including a back up device.   Have pneumothorax decompression needles and chest tube kits available.   Have a central line and IO kits ready.  Have a thoracotomy tray in the room as well.  Then get other physician and nurse help assembled.

 

When the patient arrives:  Get an airway, needle decompress or place chest tubes bilaterally,  make sure you have adequate vascular access.  Use bedside echo to identify pericardial fluid, cardiac activity, and intra-abdominal fluid.   Consider ED thoracotomy.   ED thoracotomy is probably only for penetrating trauma. You only do ED thoracotomy for blunt trauma if you personally witness loss of vital signs.   Dr. Omi comment: I don’t do ED thoracotomy for blunt trauma even if the patient looses vital signs in the ED.   Cardio-pulmonary arrest in blunt trauma patients is usually due to head trauma or aortic injury or massive intra-abdominal bleeding.   You can’t fix those problems with an ED thoracotomy.

 

 

*Thoracotomy Indications

 

Don’t do ED thoracotomy in a patient with multi-system blunt injuries, severe head trauma, or unwitnessed loss of vital signs.

Down sides to ED thoracotomy is risk of blood exposure to staff, neurologically decimated survivors, uses up your time and staff resources in a busy ED.

 

When you make your cut for an ED thoracotomy, make the cut at the nipple line in men and at the infra-mammary fold in women.     Dr. Lee comment: There are 4 things you can accomplish with an ED thoracotomy,  release  of a pericardial tamponade,  repair a cardiac injury/control bleeding, provide internal cardiac massage,  and cross clamp the aorta.

 

If you get ROSC get the patient blood and get them to the OR.  If you don’t get ROSC, carefully find and dispose of  all sharps, suture up the procedural wounds before the family sees the patient, and change out of any bloody scrubs before you talk with the patient’s family.

 

Paik      LVAD’s

 

 

*LVAD  Device

 

 Alarms

Low Battery indicates need to replace battery.

Low Flow Alarm:  Use your stethoscope to listen to the chest for the humming sound of the LVAD.  If you hear the humming sound that means the pump is working.  If the BP is low, try a small fluid bolus of 250ml.   If the pump device in the abdomen is warm (place your hand on the upper abdomen to feel if it is warm) or the RPM’s are >15000 then a clot within the LVAD may be present.

Driveline Disconnection:  This can cause syncope or death.  Reconnect the driveline ASAP!  If the patient is unresponsive start CPR while you trouble shoot the driveline.

 

Neuro complications: increased risk of intracranial hemorrhage, increased risk of emboli, chronic anticoagulation risks, and  risk of  ICH due to acquired von Willebrand Disease.   Acquired vWB disease is due to platelet damage from the pump.

 

Cardiovascular complications: Arrhythmias (Amiodarone is first line treatment for ventricular arrhythmias).  Suction events (pulls intraventricular septum toward the inflow cannula due to low volume.  Treat with IV fluids) 

 

GI complications:  GI bleeds are common due to anticoagulation and vWB disease.

 

Infectious complications: All portions of the device and the subcutaneous pocket for the device can get infected.  Biofilm producing organisms such as staph, candida, pseudomonas and enterococcus are the most common etiologies.

 

 

There was a discussion of LVAD patients presenting in V-Fib.  If the patient is alert and mentating, start amiodarone and contact cardiology/LVAD coordinator.  You may need to defibrillate the patient but if they are alert and mentating discuss with cardiology prior defibrillation.  

 

If the patient is unresponsive and in V-fib,  check if the pump is working by listening for humming with your stethoscope and check a blood pressure.  If the pump is not working or the patient has a seriously low or absent BP, start CPR and defibrillate the patient immediately.  While CPR is in progress you should trouble shoot the pump.  Check first that the driveline is connected.  If the pump is working and the BP is low give a 250 ml fluid bolus.  It goes without saying, but get Cardiology/LVAD coordinator help right away.  

 

Micheal Cirone comment: In unresponsive patients not in V-fib in whom the LVAD is working, consider other reversible causes of altered mental status like opioid use and hypoglycemia,  before you start CPR.   

 

 

 

 

 

 

 

Conference Notes 4-15-2015

 

If you don't see images scroll to bottom and click open in browser.

Carlson        Cardiovascular Drug Toxicity

 

Calcium Channel Blockers

CCB’s impair calcium influx resulting in vasodilation, bradycardia, negative inotropy, and decreased insulin production. 

 

CCB’s break down into 2  groups based on their mechanism of action:

Verapamil/Diltiazem block conduction.   

Nifedipine/Amlodipine dilate smooth muscle causing low BP. 

However, in large overdoses this distinction is lost.

Calcium channel blockers are highly protein bound and are not dialyzable.

Patients suffering from CCB toxicity present with hyperglycemia.  Hyperglycemia correlates with severity of overdose.   Hyperglycemia correlates with the need for pressors, need for pacing, and for death.

 

Early decontamination is a key management issue.   Andrea said that if you get a patient with a massive ingestion within the first hour, go ahead and perform gastric lavage.  This is the one overdose that she would recommend gastric lavage.  Patients can die from CCB toxicity and if you get a chance to intervene to decontaminate the gut early before the patient is symptomatic, it could be life-saving.   You should only perform gastric lavage if the patient is not bradycardica and not already hypotensive.

Activated charcoal is  also indicated and likely beneficial.

Next step is to battle the toxin.  Andrea cautioned us that with CCB poisoning, sometimes you can do everything right to fight the toxin and the patient still dies. 

 

Use a multi-modal approach to counter CCB toxicity. It is considered the most effective option.  Below are listed the indicated therapies.  You should use as many as you need to stabilize the patient.   

Calcium: Its effect is short lived.  It improves BP more than heart rate.  1 ampule of Calcium Chloride or 3 ampules of Calcium chloride as a single dose.  Give 1 dose every 20 minutes.  These are relatively large doses of calcium.   After 3 or 4 doses check an ionized calcium level with an ABG.

 

Atropine is the initial therapy for bradycardia.

 

Pressors: Andrea’s first choice would be norepinephrine.  Epinepherine would be next.  Vasopressin may be a reasonable adjunct due to it’s effect of increasing intracellular calcium.   There are a couple of case reports touting the combination of vasopressin and norepinephrine.  Andrea felt this combination was a reasonable option.   Harwood comment: Vasopressin is not a stand alone option for a pressor. You also need to use norepi or epi in addition to vasopressin.  Andrea agreed.

 

Glucagon can be tried at 5mg over 30 seconds.  If the initial dose is not effective,  try 10mg over a couple of minutes.  If either dose results in improvement,  start a glucagon drip.  If no effect, move on to other modalities.

Pacing is indicated for symptomatic bradycardia or conduction block.

 

Amrinone is another reasonable modality

 

High dose Insulin/Glucose has an inotropic effect by delivering metabolic fuel to a carbohydrate-dependent myocyte.   High dose insulin should be the initial therapy for calcium channel blocker overdoses.   Give 1 unit/kg of insulin as a bolus then start a drip at 0.5 unit/kg/hr.   You can titrate up on the drip.  Supplement glucose and potassium as needed.

 

Intralipid is also a recommended therapy

 

ECMO  may be an effective advanced therapy.  Contact your intensivist early to start the ball rolling for a severely unstable patient who is not improving with the above multi-modal approach..

 

Disposition: For sustained release CCB  ingestions and amlopdipine ingestions admit the patient for 24 hours.  For all other immediate release CCB ingestions, patients can be observed for 6 hours and if asymptomatic can be medically cleared.

 

Jamieson        Eye Trauma

 

 

Case 1.  30yo male was transferred to ACMC with a left orbital floor fracture and elevated IOP.   On arrival at ACMC, Patient had large subconjunctival hemorrhage.  No hyphema.  EOMI.  Visual acuity was normal.  CT showed small hemorrhage in retro-orbital region and an orbital floor fracture. Tonopen measurement of eye pressure at ACMC was normal.  Trauma service thought that that probably the eye pressure was measured in the area of the subconjunctival hemorrhage and the outside hospital get a falsely elevated pressure.  You have to take the tonopen measurement on the cornea not the sclera/conjunctiva.  

 

Retrobulbar hematoma if large can develop into an ocular compartment syndrome.   Treatment for ocular compartment syndrome with decrease in visual acuity is emergent lateral canthotomy.

 

 

*Occular Compartment Syndrome

 

 

Case2.  21 yo female presents with facial injuries due to air bag deployment.   Patient had left periorbital hematoma and a hyphema.   Visual acuity was impaired in the left eye. 

Hyphema is associated with other eye injuries including ruptured globe.

Treatment of hyphema includes daily pressure monitoring, limited physical activity,  head elevation, steroid drops and beta blocker drops.  Get ophthalmology  consult while patient is in the ED.

 

*Hyphema

 

*Hyphema Grading System

 

Case 3.  34 yo male brought in by EMS with GSW to right face.  Patient can’t open right eye and has no vision with right eye.   CT shows bullet had obliterated the orbit. 

All the following can indicate a ruptured globe:  decreased vision, teardrop/deformed pupil,  hyphema, subconjunctival hemorrhage, intraocular contents protruding from eye, and  positive seidel’s test.

 

 

*Positive Seidel’s Test

 

*Ruptured Globe

 

If you suspect ruptured globe, give vancomycin and ceftazidine, shield the eye and call ophthalmology for surgical management.

This patient had an unsalvageable eye.  His injured eye was enucleated to avoid sympathetic ophtalmia.  This can occur days to years after trauma.  It is uncommon, but is thought to be an autoimmune mechanism that causes intraocular inflammation of the uninjured eye. 

 

Harwood comment: Hyphemas tend to settle into the lower portion of the anterior chamber due to gravity.  You need to limit the patient’s activity and elevate their head to allow the blood to remain in the inferior portion of the anterior chamber.  This will keep as much of the anterior chamber clear as possible.

Lambert comment:  Topical steroids are used to decrease the incidence of adhesions within the anterior chamber.

 

 

Lambert    Occular U/S

To perform ultrasound of the eye, have the patient close the affected eye.  Place a lot of gel on the eyelids of the affected eye.  Use the linear probe on the closed, gelled eye. 

 

*U/S image of eye

 

The retina is attached in the area of the optic nerve and ora serrata.

 

*Ora serrata

 

*Retinal Detachment

 

Retinal detachments can be either with the macula still on or with the macula off.  Tell the ophthalmologist that the macula is on.  Some ophthalmologists feel that if the macula is off, the detachment is no longer an emergency.  So if you want the ophthalmologist to deal more urgently with the situation, say “macula on”

 

Vitreous detachment can look like retinal detachment but the vitreous is not tethered at the optic nerve or ora serrata.  So the detachment may include these two areas.  Differentiating this from a retinal detachment by ultrasound may be beyond the pay grade of an ER doc.

 

 

*Vitreous Detachment

 

*Vitreous Hemorrhage

 

 

*Globe Rupture

 

 

 

*Elevated Intracranial Pressure Measurement by Measuring the Optic Nerve.  You measure 3 mm from the retina.  If the nerve is more than 5mm, it corresponds to an ICP >20.

 

 

Lambert     Soft Tissue Ultrasound

 

 

*Cellulitis

 

 

*Abscess with some surrounding cobblestoning

 

 

*Pyomyositis is basically an abscess in the muscle.  These should go to the OR for drainage.

 

 

* Achilles Tendon Rupture

 

 

*Baker’s Cyst  (note the comma shape)

 

 

*Supraspinatuous Tendon Rupture  (90% of rotator cuff tears are due to supraspinatous tear)

 

*Enlarged Lymph node,  not hypo-echoic outer ring and hyper-echoic center.  Well circumscribed outer edge.

 

Ultrasound Workshop

 

 

 

 

 

Conference Notes 4-8-2015

If you don't see images, scroll to bottom and click on "open in browser"

Gore      Orthopedic Fracture/Dislocation Description

 

*Bennett (2 syllables, 2 fragments)

 

*Rolando Fracture (3 syllables, 3 fragments)

 

 

*Mallet finger

 

 

*Lunate/Perilunate dislocations

 

 

*Montaggia Fx/Dlx

 

 

*Galeazzi   Fx/Dlx

 

 

*Pediatric Elbow Ossification Centers

 

*Pediatric Ossification Centers

 

*Abnormal Pediatric Elbow Fat Pads

 

 

*Gartland Classification of Supracondylar fractures

 

 

*Posterior shoulder dislocation

 

Lovell       PE or not PE     Step-wise Approach to Diagnosis

 

Step 1.  Assess the patient’s pretest probability using gestalt, Wells score, or Geneva score.   This is a critical step that gets overlooked frequently.

 

*Wells Score   4 is the score that dichotomizes likely vs. unlikely.  Less than 4 is low risk.   4-6 is moderate risk.  Over 6 is high risk.

 

 

Step 2.  Is the pretest probability high, moderate, or low?   If high, go directly  to CT or VQ.  If moderate order a d-dimer.   If it is low then use PERC rule.   If no risk then stop the work up right here.

 

 

*PERC rule.  The PERC rule research excluded cancer patients, and hypercoaguable patients.

 

Step 3.    If PERC is positive do a d-dimer.   If PERC is negative stop the work up right here.

Step 4.   If d-dimer is positive do a CT or VQ.   If d-dimer is negative you can stop.   D-dimer sensitivity may be adversely affected in patients on anti-coagulation and those who have had symptoms for more than 14 days.

D-dimer interpretation can use age adjusted cutoffs= age X10 if the patient is over 50 yo.

D-dimer is reliable in pregnancy if a patient has a low pre-test probability and PERC is positive.

Pregnancy does not by itself make a positive Wells score or PERC rule.  You can use both tools in pregnant patients.

 

Erik Kulstad comments:  d-dimer has a better negative likelihood ratio than CT PE.   You can make the case that you can use d-dimer in high risk patients as well.  However this is not universally accepted.  Elise felt high risk patients should get a CT or VQ scan.

Harwood comment:  Do a d-dimer in all the pregnant patients whom you are considering  doing a CT.  The dimer is good enough to rule out PE and you will save a good number of pregnant patients radiation exposure.

 

Later generation CT scanning has the lowest radiation exposure for pregnant patients and is equal to VQ or perfusion only scanning.

Don’t do CT PE in patients with severe contrast allergy, renal insufficiency, multiple myeloma, or paraproteinemia.

 

Harwood comments:  Beta blockers can blunt tachycardia and obscure the diagnosis of PE. Travel is an independent risk factor for PE.  Testosterone and estrogen are also risk factors for dvt/pe.   VQ scan is not equivalent to the CT scan for diagnosing PE.  With VQ you frequently get results that are indeterminate.

 

Remke    Protocol for Ordering VQ studies on off-hours/weekends/holidays

 

High probability patient by Wells score goes right to VQ without a d-dimer.  You need to discuss the case with the radiologist.

 

Low or moderate probability patients get a d-dimer and venous dopplers of the lower extremities.  If lower extremity dopplers are negative and dimer is positive then page the VQ technician to order the test.  You don’t need to discuss the case with the radiologist.   If there is enough isotope at the hospital,  the technician will do the scan at that time.  If there is no isotope left for that day, the VQ will be done first thing the next morning.

 

Elise comment:  You only have to discuss the case with a radiologist if you do not order a d-dimer.  If you have a positive d-dimer and negative ultrasounds you don’t need to contact the radiologist.  You can just contact the nuclear medicine technician.

 

 

Parker     ED Handoffs to Observation Service

 

Case1.  75 yo female with weakness and somnolence. Multiple co-morbidities.  Work up in ED found a UTI.   Patient received IV antibiotics in the ED.   Plan was to put patient in observation rather than full admission.  Patient’s family arrived later in ED course and informed ED team that patient had been hypercapnic previously.  An ABG was ordered by the ED team just prior to signout. If  the ABG showed hypercapnea, the patient would be a full admit.   If the ABG was normal, the patient could go to observation.    The ABG showed hypercapnea.  However, the patient still was sent to the observation unit.  The patient spent multiple days in the hospital and was not a suitable candidate for observation.  Multiple issues at this transition of care were considered during this lecture.

 

Protocoled handoff processes decrease errors.  A standardized handoff tool has been shown to be useful in decreasing missed information.

Following up on patients you have received in handoff is critical to avoid errors.

Patients inappropriately placed in the OBS Unit can take up a bed for days.  That OBS bed then is not available for another patient who is appropriate for observation.

 

 

Meyers    Trauma Lecture   Cricothyrotomy

 

Patient with multiple blunt trauma was flown in by the helicopter.  Patient had a cricothyrotomy placed in the helicopter.  In the ED, the Trauma team had concerns about the stability of cricothyrotomy.   Their initial approach was to attempt was to place an endotracheal tube.  They took a look with the glidescope.  There was very poor visualization due to blood and edema. 

 

A lot of faculty comments:  You could have placed a bougie through the tracheostomy tube and then achanged out the tracheostomy tube for an ET tube over the bougie through the cricothyrotmy incision.

 

The Trauma team was able to place a bougie through the cords under direct laryngoscopy.  This was verified using a fiberoptic ambuscope through the cricothyrotomy.

 

*Ambuscope

 

Faculty comments:   You could use the ambu scope through the cricothrotomy incision to verify the cric is in the appropriate position.  Then Brian Febbo suggested that because the ambuscope is disposable, you can cut the handle off the fiberoptic cable and use the fiberoptic portion as a tube exchanger.   Everyone thought this was a great idea.

 

Case 2.  70yo male with GSW to neck.  King airway tube was placed in the pre- hospital phase.  The tube became dislodged in the ED.  Intubation attempts were unsuccessful.   A decision was made to perform a cricothyrotomy.

 

The only contraindication to cricothyrotomy is age.  Don’t do a cricothyrotomy in kids under age 8.

 

 

*Cricothyrotomy

Harwood comment:  In a person who does not have neck trauma the Melker technique is totally reasonable.

Girzadas comment: If you do a surgical cricothyrotomy, it should be considered a blind technique.  You have to be able to feel the cricothyroid membrane and puncture it and put a bougie through it without visualizing it.

There was some faculty discussion about whether surgical cricothyrotomy or  seldinger cricothyrotomy had an advantage.

 

Girzadas reference regarding the issue of surgical vs. Seldinger cricothyrotomy

It is unclear whether they are speaking of only needle crics or seldinger crics as well.  I think after doing a mini lit search on this topic, it is unclear if one techneique is superior over the other.

British Journal of Anesthesia 2011

Needle cricothyroidotomy had an unex-

pectedly high failure rate. It has been widely discussed as

to whether needle or Seldinger or surgical approaches to

direct tracheal access are best and it may be argued that

this project provides evidence that needle cricothyroidot-

omy has a high failure rate and therefore should be aban-

doned, particularly as surgical approaches were generally

successful (even when following failed needle cricothyroi-

dotomy). There are several reasons to be cautious about

such a conclusion. The NAP4 project specifically studied

events with poor outcomes and although we did seek

reports of all airway complications requiring emergency

surgical airway, it is possible that a dispr

oportionate number of successful rescue needle cricothyroidotomies

were not reported. Even if this explanation is not correct,

it is not reasonable to abandon the needle cricothyroidot-

omy technique without a much more robust explanation

of failures, which may have been due to failures of training,

use of inappropriate equipment, design problems with

appropriate equipment, or technical failures during use.

Examples of each of these observed in NAP4 include cepha-

lad placement of the device, use of an i.v. cannula for cri-

cothyroidotomy, mechanical failures of a Ravussin

cannula, and successful passage of a fine bore needle fol-

lowed by unsuccessful (and inappropriate) attempts to ven-

tilate with a low-pressure gas source.

Emergency surgical airway is the ‘final common pathway’

for all difficult airway algorithms. While much emphasis is

placed on the choice of device and technique, there is rela-

tively little written about the decision-making process and

timing of emergency surgical airway. An anaesthetic litiga-

tion review found that 42% of 179 difficult airway cases ter-

minated in CICV.

4

Errors of technique were frequent causes

of failure, particularly failure to ventilate with a high-pressure

source when a narrow cricothyroid cannula was inserted.

36

Of equal importance, persistent attempts at intubation

occurred before rescue techniques and the authors noted

that ‘our data suggest the rescue ability of (supraglottic

airways) may have been reduced by the effects of multiple

preceding attempts at conventional intubation’ and that ‘in

2/3 of the claims where CICV occurred a surgical airway

was obtained but was too late to avoid poor outcomes’. In

NAP4, there were also cases, in anaesthesia and also in the

ICU and ED, where persistent attempts at intubation

perhaps precipitated CICV, likely led to failure of rescue tech-

niques and definitely delayed emergency surgical airway.

 

 

Navarette/Bolton        Oral Boards

 

Case1.  Infant brought in by mom with increased fussiness.  Infant had some signs of trauma on torso.  History reveals child was with his father over the weekend.  Diagnosis was non-accidental trauma with intracranial hemorrhage .    Critical actions were: identify non-accidental trauma, perform a complete exam to identify ecchymoses, obtain a CT head and skeletal survey, attempt a fundoscopic exam, consult neurosurgery and admit to the PICU.  You also need to notify DCFS.  It is important to explain to the parent your concerns about abuse and that you are going to notify DCFS.

 

Case 2. 36 yo female with abdominal distension and SOB.  HR 110,  BP 98/52, RR 25.  On history it was learned that patient was receiving fertility treatment.  UCG is negative. Pelvic ultrasound shows ascites and bilateral enlarged ovaries with cysts. Chest X-ray show a pleural effusion.  Diagnosis was ovarian hyper stimulation syndrome.  Critical actions were: Consult OB, admit to hospital, administer O2. IV fluids can also be useful. Patients may need pain control.  Patients with pleural effusions should be admitted.  There is a risk of torsion due to enlarged cysts.   You must be careful when performing an abdominal and pelvic exam to be gentle so as not to rupture the enlarged ovaries.

 

*Ovarian hyperstimulation syndrome

 

*Ovarian hyperstimulation syndrome

 

Williamson comment:  These hyperstimulation syndrome patients always are closely followed by their fertility doctors and you should call them as soon as you suspect the diagnosis. They will help you immensely.

 

Case 3.  21 yo male with shortness of breath and cough.  HR 105  BP 112/56  RR 35.  Patient had been treated with a Z-pack but had not improved.  History reveals weight loss and night sweats.  Chest X-ray shows bilateral diffuse infiltrates.   Rapid HIV testing is positive. Diagnosis was  PJP pneuonia.    Critical actions were: Obtain O2 sat, get history of HIV risk factors, Obtain CXR, give appropriate antibiotics.  LDH can be helpful making the diagnosis.

 

*PJP pneumonia

 

Elise comment:  There is an old adage: If you see a kid on oral boards they were abused.  If you see a woman on oral boards they are pregnant.  You have to make sure other children in the home are not at risk of abuse as well.

 

Harwood comment:  When taking oral boards tell the examiner what you know.  Think out loud.  It helps when you don’t know the exact diagnosis.  When you do know the diagnosis say it out loud. Explain your thinking.

 

Hart     5 Slide Follow Up

 

29yo female, pregnant.  Sent in from opthomology clinic for retinal hemorrhages. Patient had headache and blurred vision.  No chest pain or shortness of breath. Patient had history of prior pre-ecclampsia.  Initial BP was 227/153.  Labs showed leukocytes and anemia with no schistocytes.

 

Patient received labetalol without improvement of BP.  Nicardipine drip was started. High Risk OB was consulted and IV magnesium was started.  Protein/Creatinine ratio was 8105.  Protein/Creatinine ratio >0.3 is diagnostic of pre-ecclampsia.  Diagnosis was pre-ecclampsia with HELLP syndrome.

Edema is no longer included in the diagnosis of pre-ecclampsia.  It is only hypertension and proteinuria.  Protein/Creatinine ratio is a confirmatory test.

Ethics consult recommended c-section of pre-viable fetus to save mom’s life.

 

Management goal is lowering MAP 20% in the first hour and 25% by 24 hours.

 

No hypertensive medication has been shown to improve morbidity and mortality from hypertensive emergency.

 

 

 

 

 

 

 

 

Conference 4-1-2015

Kmetuk      M&M

 

71 yo female  presents with syncope and altered mental status.  BP 72/60, HR 140, RR 28,  T 35C,   O2 Sat 95%.   Rhythm strip shows Afib with RVR.  Only history available was that the patient was discharged from another hospital the day before for unknown illness.

 

Initial evaluation included a negative FAST exam.  Bedside ECHO showed a suspected enlarged RV.  Abdominal Aorta was normal.

 

On the physical exam abnormal findings included: diaphoretic skin, cold skin, pale conjunctiva, diminished peripheral pulses.  Stool was heme negative.

 

EKG was Afib with RVR.  The initial therapeutic action was to cardiovert to see if the Afib was the cause of her instability.  Cardioversion converted her to sinus but she was still altered and hemodynamically unstable.

 

Patient is next intubated.  IV fluids were given and pressors started.

ABG shows a significant lactic acidosis, HGB is 7, PO2 is low for 100% FIO2 (215)

CXR is non-contributory to the diagnosis.

 

The patient’s vitals improved with the above treatment.   Patient was placed in the Queue for CT of chest and abdomen.   The CT was delayed due to Radiology wanting a Bun/Cr prior to the study.  

 

Family arrives to the ED and relates a history of DVT, mixed connective tissue disorder, and pulmonary hypertension for the patient.    Patient also had a recent cardiac cath and a subsequent right femoral pseudoaneurysm during her recent hospitalization.   The pseudoaneurysm was treated during that prior hospitalization with ultrasound guided thrombin injection.  Patient at that time was also started on Xarelto.

 

In the ED , the CT PE was negative.   CT of the Abdomen and Pelvis showed a large retroperitoneal bleed.  

 

 

*Retroperitoneal Bleed

 

The patient was given FEIBA and the massive transfusion protocol was initiated.

Patient went to IR for vessel embolization.  In the hospital she had a stormy hospital course and eventually died. 

 

Retrospective Considerations:

  1. Earlier intubation could be considered
  2. Repeat FAST exam after sedation to get a better image
  3. More frequent re-evaluations
  4. Delay in CT scanning cost some time
  5. Massive transfusion protocol possibly could have been initiated sooner
  6. Better utilization of resources possibly could have elicited more history earlier in the ED course.

 

Harwood comments:  I am proud of how well you managed this patient.  Doing the FAST exam so early was great.  It really helped you narrow down the differential diagnosis rapidly.  Patients that are agitated from shock need to have shock corrected or they won’t become last agitated.    You guys made the right move to not give TPA early on for suspected possible PE.  That move would have meant immediate death.    It was great that you pushed for the CT despite an abnormal creatinine.  You gave blood rapidly.  I also liked that you cardioverted Afib acutely. There was a lot of great management moves here.  

 

Think about retroperitoneal bleed when patients are on anticoagulants, have coagulopathy, chronic dialysis also increases the risk,  invasive procedures and blunt/penetrating trauma are also rsk factors.

Patients can complain of back, groin, flank or abdominal pain.

Look for cullen’s sign or grey turner’s sign.  However, this patient had neither of these.

 

*cullen’s sign

 

*grey turner’s sign

 

Ct plain or with IV contrast will show a retroperitoneal bleed.   FAST has a high false negative rate.  If you are going to use FAST to identify retroperitoneal bleeds, you should do sequential FAST exams to evaluate for intraperitoneal extension.

 

Treatment is IR intervention or surgery with fluid and blood resuscitation and reversal of coagulopathy.

 

Elise comment: You guys did a great job managing this difficult case.  As a department we need  to find a way to remove the barrier of delaying CT because of Radiology waiting for Bun/Cr result.

 

Garrett-Hauser      Ethics

 

Ethics in the news:  Utah passed a law allowing the state to use a firing squad as an alternate form of execution if they are unable to obtain an appropriate supply of drugs for lethal injection. 

American Academy of Pharmacists advises that no pharmacists take part in judicial executions. 

Use of firing squads for exeution may go to the Supreme Court for a decision on their legality.

 

Supreme Court will decide this summer on the use of force against the mentally ill.   The decision involves the Americans with Disability Act.   This decision may have far reaching ramifications for law enforcement and emergency medicine.

 

In Illinois, Emergency Physicians can provide information on intoxication levels of a patient when asked by the police.  You can’t however call the police to notify the police that a patient involved in a motor vehicle collision is intoxicated.   ACEP is against emergency physicians answering police questions about intoxication.   It is Shayla’s advice that we do answer police questions. The law is unclear if an emergency physician can refuse to give info to the police.   Also the police cannot force emergency department staff to draw ETOH levels or obtain a drug screen.   These samples cannot be obtained forcibly from a patient unless it is vital to their emergency care.  If the police are pressuring you regarding any of the above situations, contact the on-call administrator and they will contact the hospital lawyer.

 

Case 1.   LVAD patient had 4 teeth pulled at dentist’s office.  Patient arrived with profuse oral bleeding.   In the ED, patient becomes unresponsive.   Patient becomes more awake and refuses intubation.   Patient was intubated anyway based on the fact that  because he was in extremis he lacked decisional capacity.

 

Case 2.  38 yo male had head injury and ETOH intoxication. He was a Trauma patient.   The patient signed out AMA.  Patient fell in the waiting room bathroom and hit his head again.  He is brought back into the ED.  He then wants to sign out AMA again.  There is the question of whether the patient has capacity to make an informed decision to sign out again.  Shayla felt that the patient does not have capacity to sign out AMA again.  She based this decision on 2 factors : No clear reason why he was so anxious to leave.  Already 1 prior bad result from his first AMA decision. 

 

Case 3.   86yo female diagnosed in the ED with pneumonia.   Family did not want patient admitted.  They were going to provide home care.   Patient gets worse in the ED.   She required O2.  Family still wanted her to go home.   If she has to stay in the hospital the family wants no info given to other family members.   Shayla made the point that if a family member is making unreasonable demands on behalf of a patient, you can ask a family member to produce the health care power of attorney. This will verify that the family member actually has the legal authority to make demands/decisions.

 

Case 4.   NH patient with history of dementia. The patient arrives in septic shock.   The nursing home physician calls the ED and states he wants patient to receive IV fluids and Antibiotics.   Patient has NH POLS Form documenting that she has opted to not be transferred to the hospital and did not want IV fluids.   Family could not be contacted.  Emergency physicians opted to send patient back to the NH to respect patient’s wishes.

Elise comment:  This patient has a desire for a peaceful death. The system is working hard to not do that and provide further medical intervention.

 

Case 5.  8 month infant with gastroenteritis and dehydration. Patient has history of congenital heart disease s/p surgery.   Child won’t take po fluids.  ED physician wants patient admitted.  Mom is refusing and angry and is leaving AMA.  Shayla discussed that there were many perspectives on this case.  Optimally the ED physician can discuss with mom what she is concerned/angry about and how that can be mitigated.  More importantly a decision has to be made regarding how ill the child is.  If the child is dangerously ill or has serious cardiac disease, the emergency physician needs to take custody of the child.  If the child is not dangerously ill, probably the child can go home with the mom with instructions to get close follow up or return to the ED ASAP.

 

 

 

Jeziorkowski/Herron     Oral Boards

 

Case1.  3yo child with 3 days of sore throat and fever.  Patient has trismus.  Exam demonstrates a peritonsillar abscess.   CT head/neck shows abscess with deep space infection of the neck.   Management includes IV antibiotics to cover anaerobes and MRSA.  Also there needs to be ENT consultation for surgical drainage.  Patient requires frequent reassessment during ED course evaluating for any developing airway concerns.

 

*Peritonsillar abscess

 

Case 2.  Elderly patient presents with headache.   Patient has physical finding of tender temporal artery.   CT head is negative.  ESR is 88.   Diagnosis is  temporal arteritis .  Management includes: document visual acuity, start steroids, consult surgery to arrange for biopsy in the next few days.

 

*Temporal arteritis

 

Case 3.   40 yo male brought in by EMS for intoxication.   Vital signs are normal.  Patient is unresponsive and has signs of head injury.

CT shows acute subdural hematoma.

 

*Subdural hematoma

Management includes: check blood sugar, protect airway with intubation, consult neurosurgery for emergent surgery, seizure prophylaxis, control ICP with mannitol.

 

Elise comments: Be sure to ask for  pediatric patients’ immunization history when taking oral boards.   Be careful not to upset kids who have a potentially tenuous airway.    Sedate every patient who is intubated, even if they are altered or intoxicated.

 

Harwood comments: Always get a visual acuity and visual fields on a patient with suspected temporal arteritis.  Check for warmth, swelling, tenderness, or knobbiness of the temporal artery.   Most temporal arteritis patients can go home unless they have impaired vision.  You want to get the biopsy done within one week.  You may get a false negative finding on the biopsy if it is done after that first week of oral steroids.   In the head injured patient with subdural, clear the neck as soon as possible so you can remove the collar.  Removing the collar may improve venous return from the head and lower ICP.  Also raise the head of the bed 30 degrees.  

 

Balogun      Resuscitation of the Unstable Cardiac Pediatric Patient in the ED

 

Case 1. 2 week old infant presents to the ED with respiratory distress.   Breathing was noted by parents to be rapid overnight.  In the morning, the patient became pale and limp.

5 things to know prior to arrival of a critically ill neonate

1.  Get the vital signs

2. What is the mental status

3. What prehospital actions have been taken?

4. History

5. Patient’s weight, so you can anticipate drug dosing and device sizing

 

5 things you want to do prior to patient arrival

1. Stay calm

2.  Get pediatric crash cart and broslow tape/pedistat app for drug dosing and device sizing

3. Order Prostoglandin E and/or dopamine

4. Call pediatric cardiology

5. Call pediatric cardiology

 

On arrival, the infant was cyanotic and working hard to breathe.   The infant was arousable only with pain.

 

First action is to assess airway, breathing, and circulation.

Gain IV or IO access.  Intubate the patient.

DDX: Bronchiolitis, Pneumonia, Sepsis, Congenital heart disease/heart failure, anaphylaxis, inborn errors of metabolism.

 

Cyanosis + tachypnea + shock+ <1 month=congenital obstructive heart disease until proven otherwise

 

Within the first month of age,  unstable  congenital heart disease is most frequently ductal dependent.   These kids will respond to prostaglandin E.

 

*Ductus arteriosus

Patent ductus blood flow is critical to survival in kids with various forms of heart disease.   The ductus tends to close around 3-4 weeks of age.  This natural closure can make a stable kid with congenital heart disease become rapidly unstable.

 

ED management:

1.Intubate early.  In this case it is critical to pretreat with atropine.  Use etomidate or fentanyl to optimize hemodynamic stability.   Use rocuronium if you need a neuromuscular blocker.

2. Keep O2 sat between 80-90%.  Higher or Lower is bad.

3.  Ventilator management: AC, TV 6 ml/kg, 30 breaths per minute, Peep 5, O2 sat between 80-90%

4. PGE is life saving in an infant <1 month with cyanosis, tachypnea, and shock.

5. Dopamine at 5mcg/kg/min should be started if shock persists after PGE

6.  Third line management is an epinephrine drip

7. Give empiric antibiotic therapy after obtaining cultures

 

Dopamine is first line therapy in infants older than 1 month with shock.  After one month they no longer have ductal dependent lesions and PGE is no longer therapeutic.

Elise comment:  Have a dosing/sizing App (pedistat) on your phone for pediatric resuscitation.  It is a huge help to cognitively offload a lot of mental work doing calculations. 

 

Navarette comment: If an infant has an umbilical stump and they are in shock, you have to consider congenital heart disease and probably give PGE.  The umbilical stump falls off usually around a month of age.

 

Harwood comment:  The side effect of PGE is apnea.  All patients who are given PGE should be intubated.

 

Kadar comment: The PICU says you will never be faulted for giving PGE in a shocky kid under one month of age.  However, you will be faulted if you don’t.

 

Residents/RLT   Town Hall Meeting

 

Separate email summary will be sent out to all residents.

 

 

Remke   Safety Lecture     Inventory

 

The inventory was revamped because the hospital had 500,000 expired products on their inventory shelves.

 

Our new model is based on the Toyota model.  It is a lean supply method.   We will have the products that we need to care for our patients, not more or less.   We moved to a bin system.  There is a tracking system for the bins.    Red means inventory needs to stock more frequently, Blue means we are using the product less than expected.  Green means our re-stocking is right on track for a particular product.

 

The locked cabinet can be entered with a Code #  (the code is the charge nurse’s 4 digit phone number).  The locked cabinet contains anoscopes, extra spinal needles, tonopen,  ENT box and doppler.  We have to be accountable to bring things like the Doppler and the tonopen back to the cabinet.  

 

Supplies are grouped into wound care, airway management, and patient care.   These subgroups are color coded as well.

 

Elise comment: We should have headings on each shelving system listing the subgroup of materials on each shelving unit. 

 

There is a catalog on the front and back rows of the shelving units that lists every item on the shelves.

 

The phone number to use to contact inventory help is 41-8271 or 41-6282.

 

Each ED patient room is restocked 3 times a day.    Inventory techs should be present at every Code Yellow and Code 44’s.   No one should be questioning residents about how they are preparing for an incoming Trauma or Code 44 patient.  If you need to open the airway box prior to the arrival of a potentially critically ill patient definitely do it.

 

Regan comment:  A better labeling system or a tablet-based system that has a catalogue of items listed in commonly used  ED doc terminology for each item would be helpful.

 

 

 

 

 

 

 

 

 

 

 

 

 

 

 

 

 

 

 

 

 

 

 

 

 

 

 

 

 

 

Conference Notes 3-24-2015

Joint Peds-EM Conference

Evidence Based Guidelines for Management of Sickle Cell Disease

 

Case #1  18yo female with SS disease with Extremity Pain consistent with prior pain crises.  No fever.  CBC with a WBC of 22 and a HGB is 7.  Retic Count is 12%.

 

General Lab Values in Sickle Cell Disease

SS patients  have HGB of 6-9   Reticulocyte  counts of 10-20   WBC 10-25

SC patients have HGB of 9-11       Reticulocyte counts of  2-10     WBC  10-25

Beta thal patients have a HGB of 6-9    Retic 10-20     WBC 10-25

 

There are no tests to rule out vaso-occlusive crisis.

Pain management guideline advises parental dilaudid or other opioid analgesic should be given within 30 minutes of triage.  Reassess pain and re-administer opioids every 15-30 minutes until pain is under control.  Consider dose escalations by 25% until pain is controlled.   Initiate a PCA in the ED if possible.   Administer NSAID’s  in addition to opioids.  There was a debate during the conference on whether ketorolac or ibuprofen was better for treating vaso-occlusive pain.  Peds faculty preferred toradol.  EM faculty prefered ibuprofen. 

 

IV hydration in the ED should be one bolus followed by maintenance fluids.  No evidence favoring either Saline or LR. Fluid administration should be somewhat cautious to avoid acute chest syndrome and atelectasis.

No indication for PRBC transfusion in  the setting of pain crisis.  No indication for O2 supplementation in non-hypoxic patients in the setting of pain crisis.

In-patients should be encouraged to do incentive spirometry  ever 2-3 hours.  In-patients should also be encouraged to ambulate.  Also topical heat therapy can be helpful for pain management.

 

Case #2   4yo SS disease patient with fever and URI symptoms and cough.  CBC with WBC of 17, HGB=7.  Retic count was 8%.   CXR shows perihilar haziness.

 

Any fever in a child with sickle cell disease is treated as a medical emergency.  Fever and pain should not  be diagnosed as vaso-occlusive crisis.

Get CBC, blood and urine cultures.  Promptly administer broad spectrum antibiotics.  IV Ceftriaxone is a common choice.   Get a CXR for any cough, tachypnea, or abnormal lung sounds.   Consult  a Hematologist for all febrile sickle cell patients.

 

Sickle cell patients with fever can be discharged if well appearing, >3yo, fully immunized, family has the ability to promptly return to ED.   Hematologist at conference said she focuses on vital signs and age to make admission decisions.

Kids who get admitted will get daily ceftriaxone.   Further work up as indicated.

A big new issue is that Medicaid patients need to have a referral from their PCP every time they see the hematologist.

Case #3   12 yo male with SS disease,  presents with fever and cough.  CBC showed WBC of 13 and HGB =6.5.  CX shows right side infiltrate.

 Risk factors for Acute Chest Syndrome are asthma and prior acute chest syndrome episodes.

Acute Chest Syndrome is diagnosed by identifying a new CXR infiltrate with any one of the following: fever, tachypnea, dyspnea, hypoxia or chest pain.  Early Acute Chest Syndrome may be subtle.  There are no specific labs diagnostic of Acute Chest Syndrome.

Admit all patients with Acute Chest Syndrome.    Floor admit for stable kids is OK.  IV Ceftriaxone, oral macrolide, pain management and supplemental O2 to keep sats over 95% are indicated treatments.  Treat any bronchospasm.   Simple or exchange transfusion should be considered.  IV maintenance fluids are indicated. Any severe cases definitely should get exchange transfusion.

Steroids are not indicated for Acute Chest Syndrome based on limited data.

Elise comment: Do you ever diagnose pneumonia in sickle cell patients or is it always acute chest syndrome?  Hematologist response: It’s always acute chest syndrome.    Elise comment: It’s important to have a high concern for these patients.  Acute Chest Syndroe can have disastrous outcomes compared to simple pneumonia.

Markers of severe illness: multi-lobar disease, increased work of breathing, pleural effusions and low pulse ox despite O2 supplementation.

Case #4  8yo with SS disease presents with right side weakness and slurred speech. CBC with WBC or 10.9, HGB =7.  CT head is unremarkable. MRI/A showed scattered multifocal areas of abnormal signal.   Patient was diagnosed with stroke.

Peak incidence of stroke is 4-8 years of age.  Transcranial Doppler can identify kids at higher risk of stroke.

Work up in ED is to obtain an urgent CT followed by MRI/MRA.  Exchange transfusion is indicated for sickle cell patients with stroke.   Hematologist stated that MRI and exchange transfusion can be done in the PICU.

A second stroke is likely without regular exchange transfusions.  Even with exchange transfusion 20% of patients will have a second stroke.

 

If you give a simple transfusion in the ED in general give 10ml/KG. This will raise the HGB by 3.

 

Health Maintenance Measures for Primary Care Providers (not ER stuff)

  1. PCN prophylaxis for kids under 5.  Over age 5 is dependent on other factors
  2. Screen for proteinuria for kids over age 10
  3. Screen for HTN
  4. Screen for retinopathy
  5. Screen for stroke risk with transcranial Doppler studies
  6. Make sure women have a contraceptive plan if they don’t want to get pregnant
  7. Scheduled immunizations

 

Herrmann  (Chief Emeritus)      How to Survive your First Year after Residency

 

Keys to success: Know your Back Up and how to utilize it.  You don’t have to make all the decisions on your own.  Louie illustrated this advice with how he managed a STEMI with the help of a cardiology consultant.  They collaborated well on the case.

 

Keys to success: Know your resources in the ED.   Trust your training.  The sick patients are the easy ones.     Louie described a case of tension pneumothorax in a trauma patient that needed a rapid chest tube placement.  His success with this patient won him a lot of street cred with the nurses and other attendings.

 

Keys to success: Stay calm in critical situations when everyone is looking to you to save a life.  Be an expert multi-tasker.   Louie described a case of a patient exsanguinating who he saved with a few key sutures. At the same time he had 15 other patients in the ED.  He was able to focus on the sick patient and stay calm despite having many other patients in the ED.

 

Keys to successs: Respect and get along with nurses and other staff.  Be confident when directing your staff.  Get an idea of who you can trust and who you need to watch a little more closely.

 

Mid-levels: Basically mid-levels are professional residents.  Most are very good.  But there is a reason you went through residency.   You will find  that uncommon stuff that gets missed by mid-levels.   It takes time getting used to signing mid-level providers’ charts on patients you haven’t seen. 

 

Keys to success: Get as much pediatric experience in residency as possible.  Community ED’s have many limitations managing kids.  So you need to be prepared.

 

Scribes are the best asset ever for an emergency physician.  They are life changing. You become far more efficient when working with a scribe.  When choosing jobs, pick the job that has a scribe if all other things are equal.

 

You definitely don’t want to work in an ED where you have to disposition your psychiatric patients on your own.  It is critical to have a crisis worker to disposition psychiatric patients because they take so much time to find an appropriate treatment setting.  

 

You have to read your own plain films.

Keys to success: Get as much experience as you can reading your own films during residency.

 *(left to right) Drs. Lovell, Herrmann, Girzadas and Harwood

McKean    Syncope EKG Pearls

 

 

*Brugada Syndrome

 

*Long QT syndrome

 

 

*WPW  Delta Waves

 

 

*Wide complex irregular tachycardia, Afib with WPW

 

 

*LGL Syndrome  Kind of like WPW with no delta wave

 

 

*Tachycardia Algorithm

 

*Hyperkalemia can cause bradycardia, av blocks, and or widened QRS

 

 

*Hyperkalemia with peaked T waves (Tall, Narrow T waves)

 

 

Risk Management Issues

 

When you are considering the following common diagnoses be sure to also consider these less common life threats:

 

Common                                    Life Threats

Acute Gastroenteritis        Consider appendicitis, CO poisoning, Toxic shock

                                                  syndrome

Renal Colic                            Consider infected stone, AAA

Pyelonephritis                     Consider infected stone

Influenza                               Consider Pyelonephritis

Viral Syndrome                   Consider meningitis,  bend the patient’s neck, look at their

                                                 feet for petechiae (Harwood comment)

 

Elise comment: The patient’s vital signs with all these above common diagnoses have to be trending toward improvement.  If not then you have to consider more serious diagnoses.

 

 

Htet/Chastain       Oral Boards

 

Case 1.   50 yo male with jaw pain due to AMI.  Patient has V-Fib cardiac arrest.  Critical actions: ACLS care, defibrillate, get post-ROSC EKG, diagnose and treat inferior STEMI, get patient to the cath lab.    Optimal care: Start heparin, ASA, Plavix and hold NTG due to suspected R side AMI.  No need to cool patient because patient was conscious after defibrillation.

 

 

*Righ Ventricular AMI (note ST Elevation is greater in lead 3 than in Lead 2.  Also patient has signs of horizontal anterior ST depression consistent with posterior AMI)

 

Elise comments:  If a patient becomes unconscious, ask first whether the patient has a pulse and what rhythm is on the monitor.  Addressing V-Fib always comes before intubation.   ACLS  algorithms are driven by  the rhythm identified on the monitor.

 

Case 2.   42 yo female presents with neck and jaw pain.  Patient has brawny induration of her anterior neck.  Diagnosis was Ludwig’s Angina.   Critical Actions :  Give steroids and antibiotics, consult ENT, prepare for airway management, admit to the ICU.

 

*Ludwig’s Angina

Ludwig’s is a clinical diagnosis.  Airway compromise is the leading complication and leading cause of mortality.  Not all patients however need intubation.  Have anesthesia and ENT on consult early.   It is not uncommon that patients can spontaneously improve with early steroids and antibiotics.   Unasyn or Clindamycin are good choices to cover oral flora, which is the usual cause.   10mg of decadron is a good starting dose for management of this illness.

 

Case 3.  27 yo male with dental/facial pain after being punched in the mouth.  Felt like he spit out chips of his teeth.   Patient has upper lip laceration through vermillion border.  Patient has a fractured tooth   Patient has exposed dentin but no exposed pulp.  So this is an Ellis Class 2 dental fracture.  Treatment of this patient required inferior infraorbital nerve block and consideration of an anterior superior alveolar nerve block.    Repair laceration with approximation of vermillion  border first.  Apply calcium hydroxide (dical) over the Ellis fracture.

 

 

*Infra Orbital Nerve Block  The infraorbital nerve and the mental nerve all line up with the pupil

 

*Ellis Fracture Classification

 

 

Negro     Wellness         Focus on Fitness

 

What is fitness?  There is not a single definition of fitness. 

Increased work capacity across broad time and modal domains is the crossfit definition.

VO2 is  possibly the gold standard measure of fitness:

Average male is 35-40

Average female is 30

Elite endurance athletes are in upper 80’s and 90’s.

Siberian huskies are 240

 

Muscular strength is the ability to exert force during an activity.  Strength is increased by resistance training.

Muscular endurance can be increased by isometric resistance exercises

Other components of fitness are coordination, agility, balance and accuracy.

Results are proportional to effort.

Progressive overload is the key.

Sleep and nutrition are just as important as exercise

Keep track of your work outs

Be consistent

Find a work out buddy

Intensity matters

 

High Intensity Interval Training is a way to gain cardiovascular benefit in short periods of time.  You get increased skeletal muscle oxidative capacity,  improved exercise performance and increased VO2.  HIIT is considered safe for persons even if they have chronic disease.

 

Max heart rate calculation = 220-AGE

 

Bret then led us in a 6 minute high intensity work out.  It was quite fun and may be a feature of conference going forward.

 

 

 

 

 

 

 

 

 

 

 

 

 

 

 

 

 

 

 

 

 

 

 

 

 

 

 

 

 

Conference Notes 3-18-2015

Sorry for the long spring break from Conference Notes, but we had Simtastic, Intraining exam, AAEM, and Oral Boards.  All great EM learning events! 

Urumov     Study Guide Environmental

 

Chilblains is due to cold exposure.  Patients can have burning/pruritic pain.  Nifedipine can be used to treat chilblains.  Itching and burning pain is characteristic of chilblains. 

Immersion foot is a non-freezing injury that causes a pale, anesthetic foot. 

Freezing injury of frostbite results in ice particles in the soft tissue and again the affected extremity is anesthetic.

 

*Chillblains

 

Seizure of a diver at depth is likely due to oxygen toxicity.   Oxygen toxicity and nitrogen narcosis are likely causes when a diver has an adverse event while at a deep depth.

 

When treating a crotalid snake bite.  Give antivenin for local progression, coagulopathy/low platelets/low fibrinogen or other lab abnormalities or unstable vitals. (Girzadas mnemonic: spread, bled, almost dead) You give crofab initially as 5 vials and keep giving until you reach an end point of improvement of local progression and improvement of lab abnormalities.  

 

Venomous snakes have fangs, triangular head, elliptical eyes, heat sensing pits, and caudal plates.

Immediate care of snake bites includes immobilizing and keeping the extremity at heart level, consider a lymphatic tourniquet (the equivalent of a rubber band around the arm).  Do not cut into the wound or attempt to apply suction to the wound or apply an arterial tourniquet.

 

*Venomous snake characteristics

 

In a submersion injury patient, if the patient is asymptomatic or just has a cough and has a normal CXR and normal pulse ox and you observe for 4 hours they can go home.  Indications for admission are dyspnea, abnormal vital signs, abnormal CXR, or abnormal pulse ox.

 

When treating multiple patients following a lightning strike, treat the unresponsive, pulseless patients first.  This is the one multiple casualty situation where you attend first to the people who are dead.  Lightning strike victims in cardiac arrest have a relatively high rate of resuscitation.

 

Envenomation by sea urchins, star fish, and stingrays: Treat with Hot water immersion of the affected limb.

Jelly fish nematocysts/Portuguese man of war: Remove tentacles, wash with sea water, pour vinegar on the area or apply shaving cream.

There was a discussion of using urine to treat jelly fish stings and urine is not effective.

Sea snakes are similar to coral snakes in that both come from the elapid family.  Both have venom that can cause paralysis and respiratory depression.

 

High altitude pulmonary edema usually occurs more than 48 hours after going to altitude.  Patients have dyspnea with minimal exertion.  Dyspnea at rest is the hallmark. Lungs are not clear on auscultation.   This is the most common killer due to altitude exposure.  Treat with nifedipine or sildenafil, and oxygen/hyperbarics

 

*High Altitude Pulmonary Edema

HACE is the most severe form of altitude illness and presents with headache, altered mental status, and ataxia. Ataxia is the hallmark.  Treat with dexamethasone, oxygen/hyperbarics

 

Motzny comments on Radiation injury:  Radiation kills rapidly dividing cells.  It is similar to radiation therapy for cancer.  The most rapidly developing cells are blood.  That’s why you look at the lymphocyte count to assess the patient’s risk following radiation exposure.  The next most rapidly developing cells are GI.  So patients with vomiting and diarrhea have also been exposed to significant radiation.  Neuro and cardiac cells are the least frequently dividing cells.  If the radiation exposed patient has altered mental status or hypotension or arrhythmias these patients have had very high radiation exposure.

 

Carlson    Oral Board Day Debrief

 

  1. Tetanus case:  Patient has masseter spasm following a dirty scalp wound.  Critical actions were diagnosis of tetanus, give tetanus toxoid, give tetanus immune globulin (1/2 around wound and ½ in the deltoid),  provide wound care with irrigation/debridement and antibiotic therapy (source of toxin needs to be eliminated), ICU admit.  You can treat muscle spasms with benzodiazepines.

 

  1. Guillain-Barre case presenting with ascending weakness and absent DTR’s.  Diagnose GBS and consult neuro and admit to ICU.  Be able to describe to patient a progressive, ascending polyneuropathy.   Performing a NIF is recommended to identify patients at risk for respiratory failure.   Some residents thought the case was botulism but botulism classically is descending weakness and GBS is ascending weakness.   Treatment for GBS is IVIG and plasmaphoresis.

 

  1. Digoxin toxicity:  Identify digoxin toxicity and give 2-3 vials of digibind.  Admit to a monitored bed.   Think about digoxin toxicity if you see a rhythm strip with a fast atrial rate (aflutter) and a slow ventricular rate. 

 

*aflutter with AV block

Be cautious giving any calcium in a patient with hyperkalemia from digoxin toxicity.  Digibind will lower the potassium level in digoxin toxic patients. 

 

  1.  Patient seizing from leaking body packs of cocaine in his GI tract.  Treat seizure with benzodiazepines.  Cardiovert ventricular tachycardia. Treat blood pressure with a medication that is not a beta-blocker.  Andrea felt nicardipine would be the best choice.   Lower temperature with arctic sun cooling device or other hyperthermia management.  Arrange an emergent  laparotomy to remove the packets.  Whole bowel irrigation or endoscopy is not adequate in a body packer with leaking packets.

 

  1. Lunate dislocation.  Patient had a median nerve paresthesia.  Requires orthopedic referral for open reduction.

 

*Lunate dislocation  Note that the lunate (yellow outline) is not in line with the capitate and radius. 

 

  1. Septic joint in a pediatric patient.  Diagnose septic joint, get ultrasound to identify fluid in the joint,  arrange for emergent hip aspiration, and give IV antibiotics to cover staph.  Treat pain adequately in the child.   Septic joint fluid count should have >50,000 wbc’s.   A gram stain and culture of the synovial fluid should also be done.  Synovial fluid cultures are positive in only 50% of cases.
  2. Acute Chest Syndrome in a sickle cell patient.  Treatment included appropriate pain medication, IV antibiotics, O2 supplementation and IV fluids. Checking UCG was also appropriate.  Exchange transfusion was not a critical action but it is indicated for acute chest syndrome. There was a discussion among faculty about the importance of doing a physical exam despite knowing the diagnosis of the case.  The point was made that the new e-Oral format for the ABEM Oral Boards can lull an examinee into not doing an exam because you have video and avatar stimuli in the case.  However,  you have to be disciplined to  do a full exam in addition to the visual stimuli you receive in the case.   Harwood and Samir Patel comment:  Sickle cell patients in blast crisis look acutely ill with marked tachypnea and severe anemia.  Also some recent studies show that a reticulocyte count is not cost effective.   So you can make a good case in 2015 that you don’t need routine retic counts in patients with a HGB level that is stable compared to previous levels.
  3. GSW to head and chest.  Patient has a sucking chest wound.  Management required RSI intubation, 3 sided occlusive dressing on chest wound,  chest tube placement, IV  fluids and autotransfusion or banked PRBC transfusion, and rapidly get patient to OR.  Many people did not consider autotransfusing patient’s  blood output from chest tube.  Don’t take unstable patient to CT scanner prior to going to OR.

 

Burns     Trauma Lecture

 

Case 1. 54yo male injured while jumping from roof of burning building.

Consider inhalation injury if someone was trapped in a closed space for >10 minutes.

Should the patient be intubated?   ABG showed a PO2 of 62, CO level is 9.6%

Patients voice was becoming more hoarse.  So set up was begun for elective intubation using video laryngoscopy.   Patient had airway edema but was successfully intubated. 

Patient remained intubated for 30 hours after transfer to burn center until edema improved.  Pt was successfully extubated.

 

Elise comment: People who smoke while on oxygen will sometimes suffer flash burns of the face.  These patients won’t need intubation because there is not a risk of airway edema.   These patients  are not the same as patients who were in a closed space fire.

Trauma attending comment:  If you feel the need to intubate a burn patient, do it.   I have never regretted electively intubating a burn patient at risk for airway edema.

 

Case 2. 47yo male suffered an electrical shock when his truck trailer came in contact with a power line.

 

New Burn Nomenclature

Superficial=First degree

Partial thickness, superficial and deep=2nd degree of varying severity

Full thickness=Third degree

 

 

*Parkland Formula

Harwood comment: All the recent data on saline shows that it causes acidosis in critically ill patients when used in large quantities.  So use LR when treating burn patients with the Parkland Formula.

 

*Rule of 9’s  and Patient’s Palm Method

 

 

Girzadas comment: Succinylcholine is safe when acutely intubating a patient who has just been burned.   Succinylcholine is contraindicated when intubating a burn patient 3-7 days after injury due to the risk of hyperkalemia.   So if you have to intubate a burn patient in the ICU or burn unit use rocuronium.

 

Update tdap because burn wounds are extremely prone to tetanus.

Asymptomatic patients following low voltage shock who have a normal EKG can go home.  Pregnant patients have a risk of miscarriage and should have fetal monitoring if the fetus is >20 weeks.  In kids with oral commissure burns, you need to advise parents that patients can have severe labial artery bleeding about a week after injury.  

 

Harwood comment: In pregnant patients who have been shocked I would get an ultrasound to check fetal cardiac activity in every patient, even in those less than 20 weeks.  

Lovell comment: Dry clean sheets are adequate dressing for a patient who is being transferred to a burn unit.

 

*Burn Unit Transfer Criteria

 

Naik   New Adult Telemetry Policy

 

Limited telemetry bed availability has lead us to more closely monitor which patients are admitted to telemetry.  This also corresponds with the Choosing Wisely Campaign Nationwide.

 

A big change is patients with chest pain and a normal EKG and a negative troponin don’t need cardiac telemetry. They don’t even need medical telemetry. 

Known stable afib does not need cardiac telemetry.

Patients with AICD/Pacers with non-cardiac problems don’t need cardiac telemetry.

 

Cardiac telemetry is for cardio-centric diagnoses. 

Medical telemetry is for medical-centric diagnoses.

They are not different levels of care.  They are similar levels of care but designed for different categories of patients.

Having a telemetry monitor generally does not equal a higher level of care.

Physician judgment can trump the guidelines as needed.

 

Girzadas example: Patient with history of CHF and AICD/Pacer with vague chest pain, paced EKG, and negative troponin. Should this patient be on telemetry?   Trushar  reply: This patient could actually go to unmonitored medical bed.  If physician judgment  feels the patient should be in a monitored setting, that is acceptable as well.

 

Jeziorkowski      Case Presentation

27 yo female with heavy vaginal bleeding.  No abdominal pain. Patient had a history of asthma.  Vital signs were normal.  Vaginal exam showed closed cervix with a moderate amount of blood in vaginal vault.  HGB was 6.

During her ED course, the patient spontaneously becomes encephalopathic, agitated with focal weakness.

 

CBC showed thrombocytopenia and shistocytes.  The diagnosis was TTP.  Treatment of TTP is plasma exchange.  Steroids are also indicated.

 

Indication for plasma exchange is low platelets and hemolytic anemia.

 

*Pathophysiology of TTP.   TTP limits the functioning of AdamTS13.  Dysfuntion of AdamTS13 leads to increased von wilibrand factor  activity and resultant platelet clumps. These platelet clumps decrease platelet levels and can cause stroke or renal dysfunction.

 

Patient improved with plasma exchange and steroids.  

Additional point: If the patient has a fever over 102F that speaks against TTP.  It is much more likely to be sepsis.

 

 

 

Conference Notes 2-11-2015

Paik/McDermott     Oral Boards

 

Case 1.   30 yo male patient presents with flu symptoms/cough for a few days.  He has a fever in the ED.   CXR  was normal.   Social history reveals that the patient is a laborer at a farm and was working moving hay.  CT chest shows no PE but patient has diffuse bilateral ground glass opacities.   Diagnosis is hypersensitivity pneumonitis (Farmer’s Lung).   Hypersensitivity pneumonitis is due to lung inflammation from inhaled antigens.  Common organisms are actinomyces and aspergillus.  Treatment is systemic steroids and avoidance of inciting antigens.   Long term, this  can lead to pulmonary fibrosis.

 

Case 2. 45yo male with headache and encephalopathy.   HR=115.  Pulse ox=94%  other vitals are OK.    Dexi=100.   Patient’s wife states the patient had been refinishing a bathtub in the bathroom just prior to the patient developing his current symptoms.  It was identified that the patient was using a varnish remover containing methylene chloride.   Patient has burns on his hands from the varnish remover. Methylene chloride gets metabolized to Carbon Monoxide.   It can lead to  delayed and prolonged CO level increases.   Methylene chloride and CO are both  neurotoxic and can cause encephalopathy.  Critical actions:  Patient was decontaminated with water shower.  CT scan of head was obtained but showed no acute abnormality.   CO level was 18%.  100% oxygen is all that is usually needed to treat the increased CO from methylene chloride.  There is no specific antidote.  Hyperbaric O2 can be used if the patient seizes or is in a coma.  Admit any patient with CNS or respiratory  symptoms.  There have been a few deaths recently in people refinishing bath tubs.

 

Case 3.  42yo male with bilat eye pain.  HR=110.  Vitals are otherwise normal.  Patient woke from sleep with bilateral eye pain.  Social history reveals that patient is a welder and did not wear eye protection at work the day prior.  Diagnosis is UV keratitis.   VA=20:40 in both eyes.   Treatment is supportive with pain control.  Pain  from UV keratitis can be severe.  The damaged cornea regenerates in 3 days.  Erythromycin ointment is suggested to act as a lubricant and antibiotic prophylaxis.  Evaluate the patient for FB, consider CT of orbits if they were working with metal.

 

*Welder’s (UV) Keratitis

 

There was a discussion on the use of topical anesthetics for UV keratitis.  Harwood felt that it was reasonable to give the patient a bottle of topical anesthetic for home pain relief.  He felt that the injury was self-limited and there was very little risk of prolonged inappropriate use of the topical anesthetic that could result in eye injury.   Elise and Christine said that if there was the ability to give a patient a very limited amount of diluted topical anesthetic for home use they would give that.   Other faculty members had some reservations about giving topical anesthetics.

 

Sandeep Jauhar          Guest Lecturer at IM Grand Rounds

 

Dr. Jauhar started out by discussing how he became disillusioned about the practice of medicine:

Physicians are not getting emotional sustenance from their practice.  

Physician debt drives many physicians to moonlight/work orethan they want to.  

The speaker discussed how in certain settings, referral patterns and diagnostic testing don’t help patients but instead generates income for physicians.

There is a crisis of confidence in the medical profession.   Medicine has become just another profession.   Doctors are insecure and not enthusiastic about their profession.    Many doctors would not encourage a young person to become a physician. 

It is alarming that physicians have such a negative self-perception and that society has such diminished regard for physicians.

 

There are 3 types of workers:

Knights:  Workers who strive to make life better for others.

Knaves: Workers whose only goal is to maximize profit

Pawns:  Passive workers who follow the rules of their work place and are not that autonomous

Doctors can fit into these three categories as well.

In the 1970’s doctors had transformed in society’s eyes from knights to more like knaves.  

Since the 1980’s,  physician income has diminished despite doctors seeing many more patients and carrying increasing debt from medical school.  Add to that issues of litigation and loss of autonomy, doctors are generally unhappy.   This unfortunately is leading to a shortage of doctors and poorer patient satisfaction/happiness.

Very few patients now have any long term, personal relationship with their doctor. 

Other professions such as lawyers and teachers are also unhappy.   Physicians however were always the noble exception to the waning idea of independent professionalism.   The time frame for this exception for medicine has ended. 

Dr. Jauhar than discussed how he won back his love of practicing medicine:

Go back to how you thought about medicine in medical school and residency.

Stay true to your beliefs.

Treasure the human experiences you have as a physician.

He found that he could experience much joy in the practice of medicine again.

 

Girzadas           In-Training Exam Zebras

 

I will send out a PDF of the lecture to all the residents.  

 

Jeziorkowski       Safety Lecture: Central Line Complications

 

Cardiac Complications

ž Ventricular dysrhythmias and bundle branch block

  • •       Cause by direct stimulation of endocardium
  • •       Prevented by limiting depth of guide wire and catheter insertion to less that 16 cm

 

Vascular Complications

ž Arterial puncture issually recognized by pulsatile blood flow and red appearance of blood

  • •       Can use US to confirm venous placement
  • •       Can get ABG and blood gas from your sample to compare
  • •       Can transduce the pressure and observe wave form

 

ž If a catheter/cordis is placed in the carotid, it should be left in place and vascular surgery consulted.  If you pull the catheter/cordis out immediately after placement there is a 6% chance of stroke.

žIf a catheter/cordis is placed in the femoral artery, you can pull that line.  Injury to the femoral artery can be contained with direct pressure for 15 minutes

 

Pulmonary Complications

ž Air bubbles noted as you are aspirating the syringe may mean you violated the pleural space

ž Always get confirmatory chest x ray for subclavian and IJ lines. Subclavian lines have the highest risk of complications.

 

Guide wire Complications

ž IR can remove a  lost guide wire

ž Guide wire complications are usually due to distractions occurring during the procedure.

 

We then discussed potential safety improvements when placing central lines in our ED.  A few that seemed fruitful were: Residents turning off their phones or handing over their phones to an attending during central line placement.   Putting up a stop sign on the room door so that no one interrupts the procedure. Having a procedure cart that can be brought into the room that has all the necessary materials and a stop sign that  could be placed in the doorway to limit interruptions.

 

 

 

 

 

 

 

 

 

 

 

 

Conference 2-4-2015

Airway Day

 

Patel      Approach to the Difficult Airway

 

“Prior Planning Prevents Piss Poor Performance”

Have a plan for your intubations before you start giving drugs.  Specifically have an initial approach laid out and 1-2 back up/rescue plans if that initial approach fails.

 

4 Dimensions of a difficult airway

1.    Difficult BVM

 

*MOANS

Pregnant women in the third trimester commonly have an edematous airway

 

2.      Difficult Laryngoscopy

 

*LEMON

 

*332 Rule=3 fingers in an open mouth, 3 fingers from the hyoid to the chin,

2 fingers from thyroid notch to hyoid.  If the patient does not fit these measurements, expect a difficult airway.

 

*Mallampati score

 

If the patient has a mallampati score of  1 you can pretty much feel comfortable that you will get a decent view of the cords.  Any other mallampati score is a crap shoot. With mallampati 2-4 scores you can get anywhere from a class 1-4 view of the cords.

Asian patients tend to have difficult airway

3.      Difficult Cricothyrotomy can be assessed by visual inspection of the neck

 

Prior to intubation it is critical to pre-oxygenate the patient to saturate the hemoglobin, create an oxygen reservoir in the lungs and de-nitrogenate the lungs.

To optimize pre-oxygenation,  use a tight fitting mask and open the oxygen wall device to as high as it will go.  Wide open O2 provides a higher O2 concentration  than 15 liters/min.  In addition, keep a nasal cannula on the patient using a second oxygen wall device set at 15liter/min during the entire intubation procedure to provide passive oxygenation during intubation.   You can also use very high flow oxygenation cannulas to provide even more passive oxygenation.  

Pre-oxygenation should be done with the patient sitting up.  It improves the effectiveness of  preoxygenation due to less ded space and less weight on the thorax.

 

Cricoid pressure (Sellick maneuver) tends to displace the airway laterally.   It can compress the airway and distort the laryngeal view of the intubating physician.  There is controversy whether this maneuver is helpful/indicated and it has tended to fall out of favor. 

 

BURP/External Laryngeal Manipulation maneuver may improve the laryngeal  view.  The intubating physician moves the larynx into optimal view and the assistant holds the larynx in that position while the intubating physician places the tube.

 

 

*ELM   External Laryngeal Manipulation

 

Line up the external auditory meatus and sternal notch in all patients prior to attempting intubation.  This positioning was initially recommended for obese patients but probably is useful for all patients.

 

*External auditory meatus to sternal notch position

 

Use Ketamine to perform DSI (delayed sedation intubation).  This is sedation with preservation of respiratory drive to optimize oxygenation during intubation.  After ketamine, give 2-3 minutes of Bipap prior to attempting intubation.

 

Lovell      Airway Devices

 

In a high stakes, stressful airway scenario  you have the best opportunity for success using a device that feels familiar to you.   That’s why video laryngoscopy devices are our usual go-to rescue devices.  They have a very similar mechanical use and feel to direct laryngoscopy.

 

Supraglottic devices like an LMA can serve as great rescue devices because they are fast and can be done blindly.  They are not definitive airways because they do not provide a tube through the cords.   However, they can serve as a bridge device until you are able to pass a tube through the cords.

 

This is what you need in your armamentarium to manage airways: Bougie, LMA, direct laryngoscope, video laryngoscope,  fiberoptic/COMS scope,  and surgical airway technique.   You need to have access to and be facile with one device from each of these categories to be fully prepared to handle all potential difficult airways.

 

There is some controversy about teaching direct laryngoscopy to residents when we have video laryngoscopy available.  We (ACMC EM) still think direct laryngoscopy is important for current residents in training.  There can be instances when the video laryngoscopic device either is not available or is working sub-optimally due to fogging or blood/secretions in the airway.

 

For cardiac arrest, 2010 ACLS guidelines advise initially placing an LMA in the airway so as to not interrupt chest compression.  Proceed to intubation after you obtain ROSC if the patient remains unconscious.

 

Be sure to use airway devices (bougie, video laryngoscopy, fiberoptic/COMS) on easy airways when working in the ED so that you are  comfortable using all these advanced devices when you need them for a difficult airway.

 

When doing a cricothyrotomy the biggest error is waiting too long to start making the cut.   When you have a difficult airway, start prepping the neck as the intubating physician is working is attempting to visualize the airway  Be ready to start cutting as soon as the intubating physician identifies that intubation is not possible.

 

Age cutoff for cricothyrotomy is 8 years.  Below that age, you will need to do a trans-tracheal jet ventilation.  

 

Elise walked us through the commonly used advanced airway devices and discussed their use in specific cases.

 

Airway Workshop 

The Airway Hands-On Work Shop was fantastic!  Thanks to Drs. Bolton, Putman, and Tekwani for organizing this year’s event.  Thanks also to the over 20 faculty members who taught the workshop.  Thanks to all our faculty members who covered the ED during the Work Shop!